You are on page 1of 45

...

ANATOMIC PARTIDEMO : Purchase from www.A-PDF.com to remove the watermark A-PDF Merger SCIENCES

APRIUDECEMBER 1979

1.

Those papillae of the tongue which are the largest, the least numerous, have many taste buds and are associated with the ducts of von Ebner's glands are

7.

The degree

of mineralization is approximately that of

of normal cementum

1. " 3. 4.

fo!iate. filiform., fungiform: circumvallate.

1. 2. 3. 4.

bone. dentin. enamel. cartilage.

8.

Maturation of enamel is characteri:red by a

2.

percentageincreasein inorganiccGn~(':ntand
Cytoplasmic ribonucleic acid is localized in a percen!age 1. 2. 3. 4.
increase in water.

1. granular endoplasmic reticulum. 2. the Gol gi apparatus. 3. mitochondria. 4. Iysosomes.

decrease in water and organic con tent. decrease in water and inert' a!e in .:>rganic
con tent. increase in water and decr~ase in organic con tent.

3.

The thoracic duct extends from the upp~,"


part of the abdomen to the base of the neck where it ends in the

9.

The nerve to the masseter passes

1. 2. 3. 4.

subclavian artery. superior vena cava. junction of external and internal jugular veins. junction of internal jugular and subclavian veins.

1. through the mandibular notch to enter the 2. superior to the zygomatic arch to en ter
3.
4.

muscle on its medial surface.

the muscle on its lateral surface. posterior to the temporomandibular joint. deep to the medial pterygoid muscle.

4.

The pillars of the fauces are formed by mucosa and the underlying

10.

1. palatoglossus and styloglossus ;nuscles. 2. palatoglossus and stylopharyngeus


, muscles.

The principal fibrous elemen ts of the periodontalligamentin adults consist chiefly of

3. palatoglossus and palatopharyngeus


4.
muscles. None of the above

5.

1. 2. 3. 4. 5.

elastic fibers. coJlagen fibers. reticular fibers.

a mixture of elasticand collagenfibers;

a mixture of elastic and reticular fibers;

The lesser omentum is a

1. peritoneal fold connecting the lesser

2. 3.
4.

curvature of the stomach and the first part of the duodenum to the liver. part of the peritoneal cavity separated from the greater sac. layer of retroperitoneal fat around the kidneys. mesentery connecting the liver to the anterior abdominal wall.

11.

Incremental lines of Retzius appear

1. as lines in the enamel running at right 2. as lines in the dentin running at right
3. 4.
.

angles to the enamel surface.


angles

6.

The uterine cavity, excluding the cervical canal, is

to the dentinoenamel junction in enamel and follow the appositional pattern. in dentin and follow the appositional pattern.

1. cylindrical. 2. wider at its junctions with the cervical


canal.

12.

The zona reticularis is the inner layer of the

3. 4.

wideranteroposteriorlythan from righ t


to left. roughly triangular in shape and
compressed an teroposteriorly.

1. 2. 3. 4.

cortex of the thymus. medulla of the thymus. cortex of the suprarenal (adrenal). medulla of the suprarenal (adrenal).

.
..

13.

The vertical dimension of the thoracic cavity is increased chiefly by contraction of the

19.

The articulating surfaces of the temporomandibular joint are covered by

1. 2. 3. 4. 5.

diaphragm. quadratus lumborum muscles. external intercostal muscles. serratus posterior inferior muscles. transversus thoracis (sternocostal is) muscle.
.

1. 2. 3. 4.

hyaline cartilage. elastic cartilage. vascular fibrous tissue. fibrous or fibrocartilaginous tissue.

20.
14. The thyroid gland is characterized by the fact that it

The muscle whose tendon loops around the pterygoid hamulus is the 1. 2. 3. 4. tensor tympani. tensor veli palatini. levator veli palatini. superior pharyngeal constrictor.

1. 2. 3. 4.

becomes enlarged when it is overactive. becomes smaller after the parathyroid glands are removed. functions as the controller of general body metabolism. causes a decrease in body weight when the person has simple goi tcr.

21.
15.
The posterior lobe of the hypophysis (pituitary gland) develops from

A small bronchus differs from a bronchiole by possessing

1. 2. 3. 4.

stratified squamous epithelium and rings


or pi ates of carti! age.
-

1. 2. 3. 4.

diencephalon. mesenchyme. choroid plexus. rhombencephalon.

16.

5. A feature which characterizes a vein, compared with its companion artery, is that the vein has 1. 2. 3. 4. a smaller caliber. an indistinct endothelial lining. a more prominent tunica medica. a more prominent tunica adventitia.

stratified squamous epithelium and no rings or plates of cartilage. stratified columnar epithelium Oil1d rings or plates of cartilage. pseudostratified columnar epithelium and no rings or piates of cartilage. pseudostratified columnar epithelium and rings or plates of cartilage.

22.

. The stimulus that initiates actual formation of enamel matrix seems to be the

17.

The common hepatic artery is a branch of the

1. 2. 3. 4.

1. 2. 3.
4. 5.

celiac artery. gastric artery. splenic artery. superior mesenteric artery.

presence of predentin. reduction of the stellate reticulum. reversal of functional polarity of the ameloblast. in ftuence of increased vascularization in the dental sac. disappearance of the cell.free zone of the den tal papilla.

18.

The transverse diameter of the pleural cavity is increased during inspiration by

1. 2. 3. 4.
5.

contraction of the diaphragm. relaxation of the scalene muscles. elevation of the arched shaft of the ribs. depression of the anterior ends ot the ribs. contraction of the external abdominal oblique muscle.

23.

The middle cardiac vein empties into the 1. 2. 3. 4. 5. left atrium. right atrium. coronary sinus. great cardiac vein. anterior cardiac vein.
-, .V ..;,~

':~f

..~

:. .:-.j}:t',.{

., ..-,'.

- -

24.

A periapical abscess of a mandibular first premolar that drains lingually would drain into

29.

Progesterone production primarily by

in the ovary is

1. 2. 3. 4. 5.

the parapharyngeal space. the retropharyngeal space. a space inferior to the mylohyoid muscle. a space superior to the mylohyoid muscle. none I)f the above.

1. 2. 3. 4.

stroma. corpora lutea. ma ture foil ides. corpora albicans. 5~ growing follicles.

30. 25.
The foramen ovale is an embryological opening between

Reversal tines which may be seen on the cribriform plate (alveolar bone proper) of the alveolar process indicate the cessation of

1. 2. 3. 4. 5.

right and left atria. right and left ventricles. right atrium and right ventricle. umbilical vein and inferior vena cava. pulmonary artery and arch of the aorta.

1. 2. 3. 4.

osteoblastic. activity. osteoclastic activity. myeloid activity. healing activity-

26.

True denticles are characterized by

31.

The majority of the muscles of the larynx receive their innervation from the

1. 2. 3. 4.

having irregular shapes. containing dentinal tubules.. being located in the coronal pulp. being larger than false pulp stones.

1. cervical plexus. 2. laryngeal plexus. 3. superior laryngeal nerve. 4. inferior laryngeal nerve.

"
27.
The sternal angle is used in locating precisely the

32.

1. 2. 3. 4.

clavicle. first rib. second rib. xiphoid process.

The basic framework or stroma of ai/lymphoid tissues except thymus consists of

".

1. reticular fibers primarily and a lesser 2. a combination of epithelioid cells and


reticular fibers. amount of collagen fibers.

3. a combination of smooth muscle and 4. some smooth muscle and trabeculae of 28.
The preganglionic cell bodies of the parasympathetic portion of the autonomic nervous system are situated in

reticular fibers.

5. collagen fibers primarily and some


elastic fibers.

collagen fibers.

1. the anterolateral horn of the grey


substance of the spinal cord in the twelve thoracic and upper three lumbar cord segments. the three cervical gangliaand second. third and fourth sacralgangliaof the sympathetic trunk. association with the nuclei of certain cranial nerves and in the anterolateral cell column of the grey substance of second, third and fourth sacral cord segments.

2. 3.

33.

The posterior belly of the digastric muscle is supplied by the

1. 2. 3. 4. .5.

inferior alveol ar nerve. mylohyoid nerve. ansa cervicalis. vagus nerve. facial nerve.

.
~

34.

Lips are derived embryologically from

40.

The fate of Meckel's cartilage is believed to be

1. maxillary and mandibular processes only. 2. maxillary, mandibular and medial nasal 3. 4.
processes only. maxillary, mandibular and lateral nasal processes only. maxillary, mandibular, lateral nasal and medial nasal processes.

1. dissolution without contribution 2. 3. 4.

to any definitive structure. dissolution with minor contribution to ossification. an active role in ossification of the mandibular body. an active role in ossification of the alveolar process.

35.

The normal percentage of neutrophilic leukocytes in a differential blood count is approximately

41.

Cells of the epithelial cuff attach to enamel or cementum by means of

1. 2. 3. 4. 5.

0.5-1 per cent. 2-5 per cent. 8-15 per cent. 20-25 per cent. 60-70 pcr cent.

1. 2. 3. 4. 5.

desmosomes. hemidesmosomes. a plasma membrane. minute smooth fibrils. a 1,000 A (100 nm.) thick pellicle.

36.

The parasympathetic fibers to the pterygopalatine ganglion come from the

42.

The branchial arches concerned in development of the tongue are 1. 2. 3. 4. 5. first and second only. first, second and third. second and third only. second, third and fourth. third and fourth only.

1. 2. 3. 4. 5.

maxillary nerve. mandibular nerve. deep petrosal nerve. glossopharyngeal nerve. greater petrosal nerve.

37.

The auriculotemporal nerve carries some fibers which are

43.

The communication between the infratemporal fossa and the pterygopalatine fossa is the 1. 2. 3. 4. 5. pharyngeal canal. pterygopalatine canal. sphenopalatine foramen. pterygomaxillary fissure. pterygomaxillary foramen.

1. secretory to the parotid gland. 2. motor to the masseter muscle. 3. afferent from the carotid body. 4. sensory to the lining of the tympanic
cavity.

38.

In normal light microscopy of striated muscle, the dark portion of the striation is caused by the presence of

44.

The principal types of nerves in the dental pulp are

1. 2. 3. 4. 5. 39.

actin. myosin. Cohnheim's fields. an intercalated disk. fibers in the endomysium.

1. 2. 3. 4. 5.

parasympath etic and afferen t fibers. sympathetic and afferen t fibers. parasympathetic fibers only. sympathetic fibers only. afferent fibers only.

Stereocilia are characteristic of the

45.

1. 2. 3. 4. S.

A section of the posterior lobe of the hypophysis (pituitary) contains mainly

epididymis. seminal vesicle. ejaculatory duct. proximal convoluted tubule. ampulla of the ductus deferens.

1. 2. 3. 4.

foil ides. alpha and beta cells. chromophobes and chromophils. unmyelinated nerve fibers.

.9

t:
~

~i ,~

46.

The papillary layer of the dermis differs from the reticular 1aycr in that the papillary layer 1. contains larger blood vessels than the reticular layer. has a few elastic fibers, whereas the reticular layer has no elastic fibers. is composed of coarse collagenous fibers, whereas the reticular layer is composed of predominately fine collagenous fibers. is more finely constructed, whereas the reticular layer con rains coarser collagenous fibers as a component.

51.

The retromandibular vein is formed by the junction of the maxillary vein and the

2. 3. 4.

1. 2. 3. 4. 5.

facial vein. middle temporal vein. transverse facial vein. posterior auricular vein. superficial temporal vein.

52.

In the human fetus, developing blood cells are found in

1. liver, pancreas and lymph nodes. 2. red bone marrow, liver, spleen and
47.
The organelle that binds and releases calcium during relaxation and con traction of skeletal muscle is a

3. red bone marrow, liver, spleen and lungs. 4. submucosa of the small intestine and
lymph nodes.

lymph nodes.

1. 2. 3. 4. 5.

nucleus. lysosome. mitochondrion. transverse tubule. sarcoplasmic reticulum.

53.

In the post-eruptive phase, a tooth undergoes primarily 1. 2. 3. 4. movements in an-occlusal direction. movements in an apical direction. movements in a distal direction. rotational movements.

48.

The temporomandibular

ligamen t is found

1.

2. 3. 4.

on the lateral surface of the temporomandibular joint. on the anterior surface of the temporomandibular joint. covering the entire posterior aspect of the temporomandibular join t. connecting the mandibular condyle to the zygomatic bone.

54.

The first 'formed portion of the dentin is

1. 2. 3. 4. 5.

tircumpulpal dentin. intertubular dentin. transparent dentin. reparative den tin. mantle dentin.

49.

The submucosa of the anterolateral area of the hard palate is characterized by 1. 2. 3. 4. 5.


serous glands only. mucous glands only. mixed serous and mucous glands. adipose tissue. dense connective tissue.

55.

Primary cementum

possesses

1. 2. 3. 4.

lacunae. lamellae. canaliculi. cementocytes.

50.

During active tooth eruption, bone formation is often seen at the base of the socket. This newly formed bone is usually in the form of

56.

Embryologically, the spinal autonomic gangliaare derived from

1. 2. 3. 4.

osteons. compact bone. radiating trabeculae. horizontal trabeculae.

1. 2. 3. 4.

the neural crest. the neural tube. cranial placodes. all of the above.

.
..

57.

The cells of the sulcular epithelium are joined by

63.

Salivary glands receive innervation

1. 2. 3. 4.

desmosomes. tonofibrils. terminal webs. hemidesmosomes.

(a)
(b) (c)

58.

The cellular organelles which may be characterized as being numerous during mitosis and serving as the cytoskeleton when present in intermitotic cells are

(d)

by way of general visceral efferent nerve fibers from the salivatory nuclei. by way of general visceral efferent nerve fibers from the lateral horns of the spinal cord. from preganglionic cells located in the midbrain. from the vagus nerve.

1. 2. 3. 4.

the Golgi apparatus. microtubules. mitochondria. nucleoli. 64.

1. 2. 3. 4. 5.

(a) and (b) only (a), (b) and (d) (a) and (c) (b), (c) and (d) (c) and (d) only

Cernen turn replaces resorbed a) b) ! c) (d) bone. dentin. enamel. cemen turn.

59.

Epithelial cells of the small intestine show surface modification known as

1. 2. 3. 4. 5.

stereocilia. the ciliary border. the striated border. the cuticular border. none of the above.

1. 2. 3. 4. 5.

(a) and (b) (a) and (c) (b) and (c) (b) and (d) (c) and (d)

60.

The palatine shelf is a medial extension of the

65.

Mucus-secretingcells are found in the (a) (b) (c) (d) (e) 1. 2. 3. parotid gland. submandibular gland. mucosa of the trachea. mucosa of the ureter. glands of the esophagus.

1. 2. 3. 4. 5.

lateral nasal process. medial nasal process. intermaxillary process. mandibular process. maxillary process.

61.

From an anatomic standpoint, an emergency airway may be establ ished most readi!y by an opening into the trachea

4. .

5.

(a) and (c) (a) and (e) (b), (c) and (d) (b), (c) and (e) (b) and (d) only

1. at the level of the jugular notch. 2. through the thyrohyoid membrane. 3. through the median cricothyroid
4. 5.
ligament. between the thyroid cartilages. None of the above

66.

Muscles forming a mediolateral sling around the angle of the mandible include the

(a) (b) (c) (d) (e) 1. 2. 3. 4. 5. 6. 7.

masseter. temporalis. medial pterygoid. lateral pterygoid. digastric.

62.

Most of the antibodies in the body which act against bacterial an tigens are produced by

1. 2. 3. 4.

T-Iymphocytes. B-Iymphocytes. macrophages. plasma cells.

(a) and (b) (a) and (c) (b)and(c) (b) and (d) (c) and (d) (c) and (e) (d) and (e)

..

67.

The fate of the epithelial rests of Malassez is that they may

70.

Which of the following are correct with respect to pancreatic islets? (a) (b) (c) (d) Islets develop with duct systems' which retrogress. The products of beta cells function by reducing blood sugar level. The products of alpha cells function by raising blood sugar level. Intrinsic stroma of islets is a network of reticular fibers, but the surrounding capsule consists of collagen fibers.

(a) (b) (c) (d)

undergo calcification. form into cementicles. become fibrous. form cartilaginous nodules.

1. 2. 3. 4. 5.

(a) and (b) only (a), (b) and (c) (b) and (c) only (b), (c) and (d) (c) and (d) only 1. 2. 3. 4. 5. 6.

(a) and (b) (a) and (c) (a) and (d) (b) and (c) (b) and ~d) (c) and (d)

68.

A sinusoidal arrangement of blood vessels is found in the

71.

What is the usual sequence of even ts in the histogenesis of a tooth?

(a) (b) (c) (d) 1. 2. 3. 4.

hypophysis (pituitary).
spleen. kidney.' ' .'"

(a) (b)
(c) (d) 1. 2. 3. 4. 5. a, b, c,d, c, d, d., a, d,c,

liver.

(a) and (a), (b) (a) and (b), (c)

(b) only and (d) (c) and (d)

Deposi tion of the enamel Deposition of the dentin Elongation of the epithelial cells Differentiation of c, d a, b b, a b, c b, a

first layer of first layer of inner dental odontoblasts

72. 69.
Which of the following statements apply to DNA?

Special visceral afferent fibers for taste are conveyed in which of the following cranial nerves?

(a) (b)
(c) (d)

It is found as a component of the nucleoli. It can be distinguished from RNA by the Feulgen reaction. In polyploidy, it is a mul tiple of the normal cell chromosome content. It is not confined to the chromosomes.

1. 2. 3. 4.

V, VII and IX VII, IX and X VII, IX and XII XII only

73.

Surgical excision of the parotid gland endangers which of the following structures in addition to the facial nerve?

1. 2. 3. 4. 5. 6.

(a) and (b) (a) and (c) (a) and (d) (b) and (c) (b) and (d) (c) and (d)

1. Hypoglossal nerve 2. Motor nerves to the muscles of


mastication

3. External carotid artery and


4.
auriculotemporal nerve Lesser occipital nerve and spinal accessory nerve

;': h

. .

74.

Where does the submandibular the oral cavi ty?

duct open into

80.

Which of the following structures passes through the foramen rotundum?

1. 2. 3. 4.

At the sublingual caruncle (papilla) At the mandibular second molar At the maxillary second molar On th~ plica fimbriata

1. 2. 3. 4.

Maxillary nerve Zygomatic nerve lacrimal artery Maxillary artery

--,

75.

Which of the following organelles is the site of protein synthesis?

81.

1. 2. 3. 4. 5.

Which of the following cells is most capable of mitotic division in the adult?

Smooth endoplasmic reticulum Rough endoplasmic reticulum Golgi apparatus Mitochondrion Centriole

1. 2. 3. 4. 5.

Fibroblast Odontoblast Nerve cell Epithelial cell Smooth muscle cell

76.

The middle nasal concha is part of what bone?

1. 2. 3. 4. 5.

Nasal Vomer Ethmoid Palatine Sphenoid

82.

lymphatic vessels drain the teeth of the maxilla directly into which of the following nodes?

1. 2. 3. 4.
..

Buccal Submental
Infraorbi tal Submandibular

77.

The jugular foramen transmits which cranial nerves?

83.

1. VII, IX and XII 2. IX, X and XI 3. IX, XI and XII 4. X, XI and XII

Of the structures seen on the apical end of a partially erupted central incisor, which of the following was formed first?

78.

Which of the following best describes an osteoclast?

1. 2. 3. 4. 5.

Primary cuticle Secondary cu ticle Stellate reticulum Stratum intermedium Inner dental epithelium

1. Cell that forms bone 2. Cell of the endosteum 3. Multinucleated giant cell 4. Osteoblast which has become surrounded 5. Cell of the periphery of bone which
forms from the fibroblasts of the periosteum by a bony matrix

84.

Which of the following is characteristic of heart muscle?

1. Nodes of Ranvier 2. Rod-shaped nuclei 3. Spindle-shaped fibers 4. Centrally-placed nuclei

79.

Which of the following structures passes through rather than posterior to the diaphragm?

85.

Pressure receptors in the carotid sinus are associated with which cranial nerve?

1. 2. 3. 4.

Aorta
Esophagus Azygos vein Thoracic duct

1. Glossopharyngeal 2. Trigeminal 3. Accessory 4. Fadal

f'

....
86. Movement of the head about its vertical axis (rotation to righ t or left) occurs at which of the following joints?

92.

Which of the following best describes cell membranes?

1. 2. 3. 4. 5.

Atlan toaxial Atlanto-occipital Spheno-ocd pital Third to the seventh cervical vertebrae All of the above

1. 2. 3. 4.

One layer of charged (polar) lipids One layer of protein on the inside and one layer of neutral lipids on the outside One layer of protein on either side of a layer of neutral lipids One layer of charged (polar) lipids on either side of a layer of neutral lipids

87.

Which layer of the skin forms the epithelial root sheaths of the hair follicle?

1. 2. 3. 4.

Stratum Stratum Stratum Stratum

corneum lucidum granulosum germinativum

93.

Which principal fiber group of the periodontal ligament runs from cementum to cementum?

88.

Which of the following is the most outstanding difference between the gingiva and the mucosa of the hard palate?

1. Apical 2. Oblique 3. 'Transseptal 4. Free gingival 5. AIveol ar cres t

,.

2. Thickness of epithelium 3. Presence of glands 4. Presence of fat

Keratinization of epithelium

94.

Each of the following structures is found in the space between the medial pterygoid muscle and the ramus of the mandible EXCEPT the

89.

The muscles of mastication receive their major blood supply from branches of whkh part of the maxillary artery?

1. 2. 3. 4.

lingual nerve. lingual artery. inferior alveolar nerve. inferior alveolar artery.

1. 2. 3. 4.

Mandibular (first) part Pterygoid (second) part Pterygopalatine (third) part None of the above

95.
90. In which of the followingmay the lamina propria attach directly to bone without an intervening submucosa? 1. 2. 3. 4. 5.
Hard palate Soft pala te Gingiva All of the above Only (1) and (3) above

Each of the following structures is derived from ectoderm EXCEPT

1. 2. 3. 4. 5.

hair. enamel. dentin. sweat gland. sa!ivary gland.

91.

The primary auditory cortex of the brain is a part of which lobe? 1. 2. 3. 4. 5. Insular Frontal Parietal Occipital Temporal

96.

Each of the following cell types secretes thc substance with which it is paired EXCEPT

1. 2. 3. 4.

Sertoli's cells--testosterone. corpus luteum--progest~rone. alpha cells of the pancreas--glucagon. chromaffin cells of the adrcnal--catccholaminc.

A-PDF Merger DEMO : Purchase from www.A-PDF.com to remove the watermark


. .

97.

Abdominal organs supplied by the three unpaired branches of the aorta include all of the following EXCEPT the

98.

The smooth muscle cell shows no striations because it has no myofilaments in its cytoplasm.

..,.

l. 2. 3. 4. 5.

spleen. stomach. pancreas. vermiform appendix. suprarenal (adrenal).

l. 2.
3. 4. 5.

Both statement and reason are correct and related. Both statement and reason are correct but NOT related. The statemen t is correct but the reason is NOT. The statement is NOT correct but the reason is an accurate statement. NEJ THER statement nor reason is correct.

Ii ii :H
, ~

;~ I:
.."
'11

. j, ii

!i
;i Ii " ji r I ,.

NATIONAL ANATOMIC

BOARD DENTAL EXAMINATION PART I ANSWER KEY APRIL/DECEMBER 1979 SCIENCES

No.

MS. 4 1
4

No. 26. 27.


28.

Ans. 2 3
3

No. 51. 52.


53.

MS. 5 2
1

No. 76.

Ans. 3 2
3

1.
2.

77.
78.

3. 4. 5. 6. 7. 8. 9.
10.
11. 12. .13.

3 1 4 1 2 1 1
3 3 1

29. 30. 31. 32. 33. 34. ,35.


36. 37. 38.

2 2 4 1 5 2 5
5 1 2

54. 55. 56. 57. 58. 59.


.' 60.

5 2 1 1 2 3
5 3 4 1

79. 80. 81. 82. 83. 84.


85. 86. 87. 88.

2 1 4 4 5 4
1 1 4 3
2

61. 62. 63.

14. 15. 16. 17. 18.


19.

3 1 4 1 3
4 2
.

39. 40.

1 2 2 2 4
2 4

64. 65. 66. 67. 68.


69. 70.

4 4 2 1 2
4 4

89. 90. 91. 92. 93.


94. 95.

i I

5 5 4 3
2 3

I .
i !

I i

41.
42. 43.
44. 45.

I .
I
!
i

20.

:i I :f

. i

21.

22.
23. 24. 25.

5 1 3 4 1

46. 47. 48. 49. 50.

4 5 1 4 4

71. 72. 73. 74. 75.

3 2 3 1 2

96. 97. 98.

1 5 3

\ : f

BlOCHEMISTRY-PHYSIOLOGY
.

1.

Iodine is primarily important in the biochemical synthesis of

6.

Administration of heparin to a human results in symptoms similar to vitamin K deficiency in that both conditions result in

1. 2. 3. 4. 5.

ACTH. thyroxin. adrenalin. calcitonin. parathyroid hormone.

1. 2. 3. 4.

2.

The vitamin that promotes the syn thesis of prothrombin by the liver is

release of lipoprotein lipase to the blood and rapid clearance of chylomicrons. an increase in bleeding time due to lack of thrombin formation. retardation of fibrinogen synthesis by the liver. elevated non-esterified fatty acids which chelate serum calcium, thus retarding blood clotting.

1. 2. 3. 4. 5.

carotene. vitamin C. vitamin K. folic acid. vitamin B12.

7.

The greatest part of CO2 carried in the blood is carried in the form of

3.

The most generally accepted mechanism of fluoride action in reducing dental decay is the result of

2. 3. 4.

,.

simple solution in the blood plasma. carbamino compound with hemoglobin. bicarbonate. carbonic acid.

1. an increase in the hardness of the tooth. 2. an increase in rcmineralization of enamel. 3. a decrease in exchange of ions between 4. 5.
body fluids and the hard tissues. a reduction of solubility of the enamel as the fluoride content increases. a decrease in carbohydrate metabol ism in the oral cavity as a result of enzyme inhibi tion.

8.

The generation of the spike or action potential is attribu ted to 1. 2. 3. 4. a rapid but brief increase in Na+ conductance.

a rapid but brief increase in K+

conductance. a rapid but brief decrease in CI- shift. equal but opposite movements of K+

and Na+.

4.

Earaches may develop as a resuf t of bloc kage of the eustachian tUbe because

the ossicles can no longer vibratc. fluid in the middle ear cannot escape. pressure on the round window distorts the basement membrane. 4. pressure in the middle ear is not equalized with atmospheric pressure. 5. blockage of the eustachian tube usually means the presence of an infection in the middle ear.

1. 2. 3.

9.

In severe diabetic acidosis, one would expect an increase in plasma

1.
2. 3. 4. 5.

carbonic acid. keto acids. cations. chloride. sodium.

5.

The primary site of action of 3n.tidiuretic hormone is on the 1. 2. 3. 4. 5.


distal tubules and collecting ducts in the kidney. afferen t arterioles of the glomeruli. thirst center in the hypothalamus. osmoreceptors in the hypothalamus. adrenal cortex.

10.

Norepinephrine is stored at the


1. preganglionic parasyr:npathetic nerv.e endings. postganglionic parasympathetic nerve endings. preganglionic sympathetic nerve endings. postganglionic sympathetic nerve endings.

2. 3. 4.

12

11.

Edema may be caused by

17.

1. constriction of arterioles. . 2. increased permeability of capillaries. 3. reduced blood pressure in the capillaries. 4. a tissue oncotic pressure that is lower
than that of the plasma.

Carbonic anhydrase in the kidney tubular cells is known to be associated with the reabsorption of

1. 2. 3. 4.

urea. chloride. bicarbonate ion. carbohydrate.

12.

Fasting for three hours leads to

1. 2. 3. 4.
13.

18.

increased storage fat. decreased liver glycogen. elevated blood sugar level. suppression of gluconeogenesis.

Toxic effects of carbon monoxide are best explained by the fact that carbon monoxide

1. depresses the rate of respiration. 2. preven ts dissociation of oxyhemoglobin. 3. competes with O2 for hemoglobin
4. 5.
binding sites. poisons oxidative cells. catalysts of the tissue

The rate of fluoride depends upon the

incorporation

into bone

1. 2. 3. 4. 5. 14.

water.mineral

ratio.

combines irreversibly with hemoglobin forming methemoglobin.

age of the individual. rate of bone remodeling or turnover. amount of ionizable fluoride in the diet. All of the above

19.

leads to 1. 2. 3. 4. 5. 20.

Hypoactiviry of the posterior pituitary gland dwarfism. cretinism. acromegaly. diabetes mellitus. diabetes insipidus.

In a parathyroid deficiency state, there is

1. 2. 3. 4.

an increase in serum calcium and a decrease in serum phosphate. a decrease in serum calcium and an

increasein serum phosphate.

an increase in serum calcium and a normal serum phosphate. a normal serum calcium and an increase in serum phosphate.

The first heart sound relates to

15.

Oxygen removal from the alveoli of the lung may be facilitated by

1. 2. 3. 4. 5. 21.

closure of the A-V valves. opening of the A.V valves. excitation of the auricles. closure of the aortic valve. opening of the aortic valve.

1. 2. 3. 4. 5. 16.

low PO2 of pulmonary artery blood. increased total alveolar surface area. increased blood flow through alveolar capillaries. all of the above. none of the above.

Removal of a molecule of water between the carboxyl group of one amino acid and the amino group of a second amino acid results in formation of a

An action poten tial in a nerve fiber is considered to be related to

1. 2. 3. 4. 5.

zwitterion. polypeptide. peptide bond. hydrogen bond. glycosidic bond.

1. 2.

3. 4.

the changed orientation of molecules in the membrane giving rise to a static potential difference. the entry of sodium ion followed by the exit of potassium ion. a breakdown of metabolic products resulting in different concentrations of potassium across the membrane. the flow of electrons across the membrane following change in membrane permeability.

22.

Augmented flow from the gallbladder during feeding results in part from

1. 2. 3. 4. 5.

a conditioned reflex. the release of enterocrinin. the release of cholecystokinin. a reflex by way of the splanchnic nerves. the stimulatory effect of bile salts by way of enterohepatic circulation.

23.

The hemodynamics

of flow in the tooth pulp are most likely analogous to those in the

29.

The absolute refractory period of skeletal muscle is an interval during which.

1. 2. 3. 4. 5. 24.

heart. lungs. cranium. liver. gut.

1. the stimulus is more than normally 2. 3. '4.


effective. the threshold is lowered. the muscle is relaxea. no Himulus is effective.

i~:;;~,:t ,:~~~ -~ ~~~}f -->'" .',:-O"


-\ >-., <

Muscle glycogen does not yield blood glucose directly but liver glycogen does because

30.

The primary function of the cerebellum is

1. 2. 3. 4. 5.

hexokinase is not present in liver. muscle cells are impermeable to glucose. muscle does not contain phosphoglucomutase. muscle glycogen differs in structure from liver glycogen. glucose-6-phosphatase is not present in muscle.

l. 2. 3. 4.

coordination of spinal reflexes. coordination of autonomic activity. coordination of voluntary movement. regulation of the intemal environment of the body.
lipotropic substance is

31.

An important

25.

A 0.85 per cent solution of sodium chloride is isotonic with respect to blood. This solution would also be

1. 2. 3. 4. 26.

isosmotic. hyposmotic. hyperosmotic. none of the above.

1. 2. 3. 4. 5. 32.

chol ine. tyrosine. hemoglobin. oxaloacetic acid. J3-hydroxybutyric.acid.

Transcription is the ce1lularprocess of making

Glomerular filtration rate may be measured by

1. creatinine. 2. :l substance fittered and secreted. 3. a substance filtered but not reabsorbed
4. or secreted. none of the above.

1. new DNA. 2. RNA from DNA. 3. proteins from amino acids by way of 4. 33.
RNA. none of the above.

Immediately following the closure of the A-Vvalves is the period of

27.

An important nutrient present in low concentration in milk is

1. 2. 3. 4. 28.

carbohydrate. calcium. protein. Iron.

1. 2. 3. 4. 34.

isotonic contraction. isometric con traction. isometric relaxation. isotonic relaxation.


required in a

A component of the coenzyme transamination process is

In the relationship between the concentrations of substrate and the rate of an enzyme catalyzed reaction, the existence of a limited value (Vmax) of the reaction rate is due primarily to the

1. 2. 3. 4. 5.

nIacin. thiamine. folic acid. pyridoxine. riboflavin.

1. exhaustion of the substrate supply. 2. saturation of the enzyme with substrate. 3. inhibition of the enzyme by the reaction 4.
products. denaturation of the enzyme at higher substrate concentrations. balance between the increase in reaction rate with increasing substrate concentrations and accelerated destruction of the enzyme at higher substrate concentrations.

35.

A marked fall in the oxygen tension in arterial blood would stimulate the receptors in the

s.

1. 2. 3. 4.

aortic arch and the carotid sinus. walls of the great veins. aortic and carotid bodies. respiratory center.

--

36.

In terms of its origin and its function, the adrenal medulla is comparable to

41.

The largest volume flow of blood through the left coronary artery occurs

1. 2. 3. 4. -. 5.

the adrenal cortex. a pregangl ionic sympathetic nerve. a postganglionic sympathetic nerve. a pregangl ionic parasympathetic nerve. a postganglionic parasympathetic nerve.

1. 2. 3. 4. 5.

during systole. during diastole. during rapid ventricular ejection. during isovolumetric contraction. between first and se'ond heart sounds.

37.

Sodium taurocholate and glycocholate are necessary for the absorption of

42.

After being absorbed from the intestine, iron is carried through the blood in the form of 1. 2. 3.4. 5. ferritin. transferrin. hemosiderin. apotransferrin. ceruloplasmin.

1. 2. 3. 4. 5. 38.

sucrose. glycerol. amino -acids. fatty acids. nucleic acids.

The rate of diffusion of oxygen across the alveolar-capillary membrane is

43.

1. directly proportional to the alveolar 2. directly proportional to the functional


diffusion surface area. oxygen tension. volume.

In humans, the site for production of ketone bodies in blood appears to be

3. indirectly proportional to the alveolar

4. indirectly proportional to the oxygen


tension gradient across the diffusion membrane. 44.

1. 2. 3. 4. 5.

the liver. the ki dney. the ad renal. the pancreas. adipose tissue.

The main function of plasma globulins is to

39.

The belief that the secretion of saliva is an active process is supported by the observation that 1. 2.
during secretion, there is vasodilation

1. 2. 3. 4.

maintain colloid osmotic pressure. provide the body with both natural and acquired immunity. act as a framework on which clot formation may occur. None of the above

within the gland.

'

3. 4. 5.

during secretion, blood flow through the gland is increased. during secretion, blood flow through the gland is decreased. secretion stops if the pressure within the salivary duct is the same as the blood pressure. secretion continues even when the pressure within the salivary duct is higher than the blood pressure.

45.

Duringexercise,muscletissueaccumulates lactic acid. As a result, erythrocytes passing through capillaries in the muscle

1. 2. 3. 4. 5.

release more CO2, absorb more CO2. release more O2. Both (1) and (3) above Both (2) and (3) above

40.

The subliminal fringe of a motor neuron pool ISa useful concept in explaining the phenomenon of

46.

The enterogastric reflex produces

1. 2. 3. 4.

direct inhibition. reciprocal innervation. central facilitation. monosynaptic conduction.

1. 2. 3. 4. 5.

an increase in gastric secretion. an increase in gastric motility. a decrease in gastric motility. a decrease in intestinal motility. an increase in intestinal motility.

47.

Section of the pyramidal tracts produces

52.

In third degree heart block, the electrocardiograph shows

1. 2. 3. 4. 5.

hypertonia. sympathetic hyperactivity. parasympathetic hyperactivity. loss of fine voluntary movements. complete loss of temperature regulation.

1. 2. 3. 4.

an increased PQ interval. an increase in the height of the R wave. an increase in the height()f th.e QRS complex. .. .. the P wave and the QRS complex arc dissociated.

53. 48.
The best criterion for the effectiveness of breathing is

The most biologically important physicochemical property of connective tissue which is regulated by its mucopolysaccharide molecules is

1. 2. 3. 4.
.

5.

tidal volume. minute volume. rate of respiration. alveolar ventilation. a careful measurement

1. 2. 3. 4.
of the dead space.

viscosity. buffering capacity. solubility in dilute acids. supersaturation with calcium ions.

54.

49.

The effect on basal heart rate of cutting successively the vagus and the sympathetic nerves in an animal reveals

A su hstance that al ters the rate of an enzymatic reaction by interacting with the enzyme at a site other than the active site is

1. 2. 3.

4.

predominance of vagal over sympathetic innervation in determining heart rate. predominance of sympathetic over vagal innervation in determining heart ratc. that effects of the vagi and the sympathetics on heart rate are equal and opposite. none of the above.

1. 2. 3. 4.

an allosteric modifier. a competitive inhibitor. a non-competi tive inhibitor. none of the above.

55.

Homogentisic acid accumulates in the urine in

1. 2. 3. 4. 56.

ketonuria. cystinuria. alkaptonuria. phenyl ketonuria.

An enzyme that is activated by epinephrine is

50.

Saltatory conduction depends largely upon the

1. 2. 3. 4.

presence of Golgi bodies in the soma. internal concentration of sodium. presence of nodes of Ranvier. presence of Schwann cells.

1. 2. 3. 4. 5.

hexokinase. glucokinase. adenyl cyclase. phosphofructokinase. none of the above.

57.

51.

Cye!ic 3', 5 '-AMPincreases the rate of glycogenolysis by

In helping to control the rhythm

v,~

respiration, the vagus nerves in the lungs are stimulated by receptors detecting changes in the

1. promoting the formation of a


phosphorylated form of glycogen 2.
phosph ory Iase.

1. 2. 3. 4. S.

stretch of the lung parenchyma. PCO2and Po of the blood. PCO2 of the a1veolarair. PO2 of the alveolar air. pH of the blood.

3. 4.

serving as a substrate for glycogen phosphorylase. serving as a precursor of 5' AMP which is a cofactor for glycogen phosphorylase. furnishing phosphate for the phosphorolysis of glycogen.

I
l

..
58.
A proportionate increase in resistance of afferent and efferent arterioles of the kidney would result in a decrease in 1. 2. 3. 4.
both filtration fraction and glomerular fiItra tion ra te. renal blood flow with no change in glomerular filtration rate. glomerular fil tratiol1 rate with no change in renal blood flow. urine ou tput.

63.

Albinism is caused by a deficiency of an enzyme involved in the metabolism of 1. 2. 3. 4.


wi.

.:::

tyrosine. arginine. histidine. methionine. tryptophan.

.'

64.

The main factors directly involved in the maintenance of systemic arterial blood pressure are

59.

The depolarization of the skeletal muscle cell membran.eby motor nerves is directly produced by the

1. cardiac outpu t, blood volume, peripheral

2. cardiac output, blood volume, blood 3. heart rate, ventricular filling, blood
4.
viscosity and peripheral resistance. plasma colloid osmotic pressure, capillary blood pressure, tis)ue hydrostatic pressure and tissue colloid osmotic pressure. viscosity and peripheral resistance.

resistance and respiratory rate.

1. shift of Na ions out of the end-plate. 2. activation of muscle contractile proteins. 3. increased concentration of neuromuscular
4.
cholinesterase. change in end-plate potential level to a critical value.

60.

Vagal stimulation in the mammalian heart has primarily a rate effect because

65.

The buffer system most important in maintaining the physiological pH of plasma is

1. 2. 3.

4.

acetylcholine has little direct effect on contractilit~, of heart muscle. there are few vagalendings distributed to the mammalian ventricles. acetylcholine is not liberated in effective amounts at vagalendings in the mammalian ventrides. sympathetic effects cancel those of the vagus.

1. 2. 3. 4. 5.

protein/proteinate. acetic acid/acetate. carbonic acid/bicarbonate. phosphoric acid/phosphate. hydroxybutyric acid/hydroxybutyrate.

66.

Pieces of dog parotid gland and of pancreas are transplanted under the skin of the donor animal. After re-establishment of a circulation, ingestion of food will result in secretion from

61.

The osmotic pressure of the fiI trate at thc end of the proximal convoluted tubule is

1. 2. 3. 4.

greater than that of plasma. much less than that of plasma. about the same as that of plasma. greater than that of the filtrate in the descending portion of the loop of Henle.

1. 2. 3. 4.

parotid and pancreatic transplants. the fragment of the parotid only. the fragment of the pancreas only. neither fragment.

67.

The products of hydrolysis of lecithin are 1. glycerol, fatty acids, phosphoric acid, serine. glycerol, fatty acids, phosphoric acid,
.

62.

Osteoblasts form the organic matrix of bone prior to calcification of the tissue. This process requires vitamins

2 3.

choline.

1. 2. 3. 4. S.

A and BI ] .
A and C. A and D. C and E. D and E.

sphingosine, acetic acid, phosphoric add, inositol. 4. glyceraldehyde, fatty acids, phosphoric acid, choline. s. glyceraldehyde, fatty acids, phosphoric acid, ethanolamine.

.
.

~
;t;~~~;' ~

68.

If the anticodon on transfer RNA is ACG, the codon on messenger RNA is

74.

Capillary diameter is directly influenced by

1. CGT. 2. GCA. 3. CGA. 4. UAG. 5. UGc.

1. 2. 3. 4.

parasympathetic nerve impulses. byproducts of metabolism. sympathetic nerve impulses. axon reflexes.

69.

Two hormones which act similarly to increase glycogen and lipid breakdown as well as cyclic AMPsynthesis are 1. 2. 3. 4.
insulin and calcitonin. glucagon and epinephrine. aldosterone and testosterone. parathyroid hormone and glucagon.

75.

Coenzymes used in pyruvate dehydrogenase reactions are (a) (b) (c) (d) (e) thiamine pyrophosphate. pyridoxal phosphate. lipoic acid. biotin. NAD+

70.

An injection of epinephrine in to the left. yen tricular wall causes an increase in cardiac output. This is an example of

L 2. 3. 4. 5.

1. 2. 3. 4.
-

(a) and (b) only (a), (b) and (e) (a), (c) and (e) (b) and (d) (c) and (d)

positive inotropic activity. heterometric autoregulation. Starling's Law of the Heart. homeometric autoregulation.

71.

The amino acid composition of a protein having an isoelectric pH of 10 has

76.

1. 2. 3. 4. 5.

no acidic amino acids. no basic amino acids. no neutral amino acids. more basic than acidic amino acids. more acidic than basic amino acids.

A person injured in an automobile accident has lost two pints of blood and is now in shock. Which of the following are likely to occur? (a) (b)
(c) (d)

72.

daily dietary allowances for Ca and P provide a Ca/P intake of approximately

The recommended

A decrease in glomerular filtration rate A decrease in the reabsorption of H2O in the distal convoluted tubule A decrease in-urine formation
A decrease in the release of an antidiuretic hormone

1. 2. 3. 4. 5.

0.5. 1.0. 2.0. 3.0. 4.0.

1. 2. 3. 4. 5.

(a) and (c) (a) and (d) (b) and (c) (b) and (d) (c) and (d)

73.

Other factors being equal, one should expect N2O-O2 sedation to be induced more slowly than normal in a person having a

77.

Which of the following is an essential fatty acid?

1. tidal volume larger than normal. 2. vital capacity smaller than normal. 3. residual volume smaller than normal. 4. functional residual capacity larger than

5. None of the above

normal.

1. 2. 3. 4. 5.

Acetic Stearic Myristic Palmitic Arachidonic

,..
~ ..

r ~

.
..'
78. Which of the following functions of bone could be considered the most prominent?

84.

In the metabolic conversion of glycogen to lactic acid, which of the following is an intermediate? 1. 2. 3. 4. 5.
Maltose Glycerol Phosphocreatine Acetoacetic acid Glucose-l-phosphate

1. 2. 3. 4.. 5.

Buffering Phosphate source Sodium reservoir Fluoride reservoir. Calcium reservoir

79.

A number of substances yield energy to the body when a phosphate group is transferred to a suitable acceptor. Which of the following compounds yields the most energy per mole? 1. 2. 3. 4. 5.
Creatine Pyruvic acid Glucose-l-phosphate Adenosine triphosphate Fructose-l,6-diphosphate

85.

Whichof the following is most likely to promote depolymerization of the ground substance? 1. 2. 3. 4. 5. Cortisone Collagenase Chymotrypsin Hyaluronidase Ammonium phosphate

80.

Which of the following is the direct precursor of urea?

86.

Where in the cell does most protein synthesis occur?

1. 2. 3. 4. 5.

Glutamic acid Citrulline Ornithine Glutamide Arginine

1. 2. 3. 4. 5.

In the nuc;leus ,Within Iysosomes . Within the mitochondria On polyribosomes In soluble cytoplasm

81.

Which of the following enzymes is involved in the termination of neuromuscular transmission? 1. 2. 3. 4. 5.


Acetylcholine esterase Methyl trans ferase Monoamine oxidase Choline acetylase Adenyl cyclase

87.

What changes in the extracellular fluid compartment are caused by prolonged sweating? 1. 2. 3.
4. A decrease in volume and an increase in

osmotic pressure

82. To which of the following avitaminoses is


the gingivamost susceptible?

A decrease in volume and a decrease in osmotic pressure An increase in volume and an increase in osmotic pressure An increase in volume and a decrease in osmotic pressure

1. 2. 3. 4. 5.
83.

A B2 86 C Niacin

88.

Which of the following statements concerning salivary gland function or its control is correct? 1.

Which of the following ions is especially required in the blood clotting process?

1. 2. 3. 4. 5.

Fe++ Na+ ea++ Mg++ K+

Epinephrine inhibits parotid amylase secretion by decreasing cyclic AMP levels. 2. Salivary flow is inhibited by the presence of dry food in the mouth. 3. Some chemicals, such as potassium iodide and mercury, are excreted in part by the saliva. 4. Mucoproteins are the most important buffers in the saliva.

h >.

.
.

89. Whichof the situations below characterizes


the exchange of chloride and bicarbonate ion between the red cell and plasma in the lungs?

93.

Which of the following reflexes is monosynaptic?

1. The passage of chloride ion into the cell


from the plasma and the passage of bicarbonate ion from the cell into theplasma. The passage of chloride ion into the plasma from the cell and the passage of bicarbonate ion from the plasma into the cell The passage of chloride ion and bicarbonate ion into the cell from the plasma so that carbon dioxide can escape The passage of chloride ion and bicarbonate ion from the cell into the plasma

1. 2. 3. 4. 5. 94.

5tretch reflex Withdrawal reflex Crossed ex tensor reflex Positive supporting reaction Labyrinthine righting re~lex

2.

3. 4.

In which of the following combinations is the name of the hormone, its chemical type and its tissue of origin correctly matched?

1. 2. 3. 4. 5.

AIdosterone-.pep ti de--pancreas Glucagon--peptide--adrenal cortex Epinephrine--steroid-adrenal medulla ACTHupo/ypeptide--adrenal cortex Vasopressin--peptide--posterior pituitary

95.

90.

Which of the following amino acids is a thio-ether? 1. 2. 3. 4. 5. leucine Threonine Isoleucine Methionine Thiolhistidine

Whichof the following is considered to be the least important in producing enamel hypoplasia in developing teeth?

1. 2. 3. 4.
5.

Rickets Prolonged vitamin A deficiency Prolonged vitamin D deficiency Fluoride intake of less than 0.2 ppm in the water Inadequate calcium intake for a prolonged period

96.

Which of the following is NOT a type of connective tissue fIber?

91.

Which of the following processes is most importan,t in preventing a decrease of more than 0.2 units in the pH of the blood when CO2 en ters?

1. 2. 3. 4.

Chitin Elastin Collagen Reticulin

l. 2. 3. 4. 5.

Urea formation following amino acid deamination NH4 + formation in the kidney Formation of ketone bodies Excretion of acidic urine Conversion of HbOz to Hb

97.

Which of the following-digestive enzymes is NOT derived from an inactive zymogen precursor? 1. 2. 3. 4. Pepsin Trypsin Amylase Chymotrypsin

92.

Which of the following is most commonly associated wi th transport of free fatty acids in human blood?

98.

The hypothalamus-pituitary complex does NO T control hormonal secretions of the

1. 2. 3. 4. 5.

AIbumin Globulin Cholesterol Sphingolipid Mucopolysaccharide

1. 2. 3. 4. 5.

thyroid. parathyroids. corpora lutea. ovarian follicles. interstitial cells of the testes.

20

.. ..
99.
An enzyme which is found in the succus entericus bu tNO T in the pancreatic juice is 100. Proprioceptors are found in each of the following EXCEPT the, 1. 2. 3. 4. 5. gingiva. skeletal muscles. pulp of the tooth. periodontal ligament. temporom~andibular joint.

1. 2. 3. 4.

lipase. trypsin. chymotrypsin. enterokinase.

.
&

NATIONAL BOARD DENTAL EXAMINATION PART I ANSWER KEY BIOCHEMISTRY-PHYSIOLOGY APRIL/DECEMBER 1979

No.

MS. 2 3 4 4 1 2 3 1 2 4 2 2 5 2 4 2 3 3 5 1 3 3 3 5 1

No. 26. 27. 28. 29. 30. 31. 32. 33. 34. 35. 36. 37. 38. 39. 40. 41. 42. 43. 44. 45. 46. 47. 48. 49. 50.

Ans. 3 4 2 4 3 1 2
- -

No. 51. 52. 53. 54. 55. 56. 57. 58. 59. 60. 61. 62. 63. 64. 65. 66. 67. 68. 69. 70. 71. 72. 73. 74. 75.

Ans. 1 4 1 1 3 3 1 2 4 2 3 2 1 2 3 3 2 5 2 1 4 2 4 2 3

No. 76. 77 -. 78. 79. 80. 81. 82. 83. 84. 85. 86. 87. 88. 89. 90. 91. 92. 93. 94. 95. 96. 97. 98. 99. 100.

Ans. 1 5 5 4 5 1 4 3 5 4 4 1 3 1 4 5 1 1 5 4 1 3 2 4 3

1.
2. 3. 4. 5. 6. 7. 8. 9. 10. 11. 12. 13. 14. 15. 16. 17. 18. 19. 20. 21. 22. 23. 24. 25

4 3 3 4 2 5 3 2 2 1 2 5 3 4 4 1 3

A-PDF Merger DEMO : Purchase from www.A-PDF.com to remove the watermark


PART I MI CROB10 LOGY-PATH0 LOGY
/

-1.

Dermatotropic

viral diseases include

6.

In children, the bones become bowed in rickets because of failure of

l. 2. 3. 4. 5.

smallpox, chickenpox and measles. hepatitis, mumps and yellow fever. impetigo, scarlet fever and scabies. rabies, herpes and encephalomyelitis. influenza, common cold and primary atypical pneumonia.

l. 2. 3. 4.

osteoblasts to develop. osteoid tissue to calcify. osteoid tissue to form. cartilage matrix to form.

2.

7.
The most common source of a pulmonary
embolism is
.

The spreading factor produced by certain streptococci is

l. 2. 3. 4. 5.

hemorrhoids. the mitral valve. the portal vein. the deep leg vein. a long bone fracture.

l. 2. 3. 4. 5.

antigen. toxemia.

coagulase. .
fibrinolysin. hyaluronidase.

8.
3. The ability of certain oral microorganisms to function as etiologic agents pf..dental caries appears to be correlated with the cap.acity of
these organisms to produce
.

Systemic lupus erythematosus, rheumatoid arthritis, idiopathic thrombocytopenic purpura and thyroiditis are examples of 1. 2. 3. 4. Arthus reaction. autoimmune diseases. anaphylactic reactions. immediate hypersensitivity reactions.

proteolytic enzymes capable of hydrolyzing salivary proteins. 2. capsules which inhibit the activity of phagocytic cells present in saliva. 3. carbonic anhydrase, an enzyme active in decreasing the CO2 content of saliva. 4. an extracellular polysaccharide dextran-like substance involved in the formation of dental plaque.

1.

9.

Hypothyroidism in children results in

1. 2. 3. 4. 5.

myxedema. gigantism. cretinism. acromegaly. diabetes insipidus.

4.

An oral disease characterized by white patches on the buccal mucous membranes which consist largely of pseudomycelia with minimal erosion of the membranes is caused by

10.

True bacteria multiply by 1. 2. 3. 4. 5. budding. sexual fusion.

1. 2. 3. 4. 5.

Candida albicans. Treponema pallidum. Entamoeba histolytica. Sporothrichum schenckii. Corynebacterium diph theriae.

binary fission.

fragmentation of mycelia. formation of sexual spores.

11. 5.
Vitamin C deficiency will

The genus of fungi most frequently recovered from healthy mucous membrane surfaces is 1. 2. 3. 4. Candida. Aspergillus. Histoplasma. Blastomyces. Coccidioides.

,.

2. 3. 4.

impair collagen formation. impede re-epitheliaJization. prevent proliferation of fibroblasts. delay resolution of pre-existing exudates.

5.

,
12.
Megaloblastic anemia may be related to

17.

1. 2. 3. 4.

iron deficiency. folic acid deficiency. vitamin K deficiency. aminopyrine therapy.

The bacterial population in the gingival sulcus or the pocket that influences the course of periodontal disease has been found to involve

1. mostly aerobic bacteria. 2. essentially a pure culture. 3. bacteria not indigenous to the oral 4.
cavity. essentially the same organisms found in the healthy sulcus.

13.

The greatest incidence of rubella associated with cardiac anomalies occurs when the infection is established

1. 2. 3. 4. 5.

18.
during the first trimester in utero. during the second trimester in utero. during the third trimester in utero. immediately post partem. None of the above

A berry aneurysm may result in 1. 2. 3. 4. cerebrovascular accident. sudden, severe chest pain. painless swelling of the leg. focus of radiolucency in a long bone.

19.

Plastic items requiring sterilization should be sterilized using 1. 2. 3. '4.


dry heat. ethylene oxide. cationic detergen ts. 70 per cen t ethanol. 5. . autoclaving techniques.

14.

Ten days after hospitalization for a large, incapacitating myocardial infarct, a 50-year-old man suddenly develops paralysis of the right side of his body. The best explanation for his brain damage is

1. 2. 3. 4. 5.

rupture of a congenital aneurysm of the circle of Willis. brain abscess from necrosis of the myocardium. detachment of a bacterial embolus from the pulmonic valve. detachment of a mural thrombus from the right ventricle. detachment of a mural thrombus from the left ventricle.

20.

When malignant cells resemble more primitive, undifferen tiated cells than those from which they arose, they are said to be 1. 2. anaplastic. metastatic. hypoplastic. metaplastic. desmoplastic.

3. 4. 5. 21.

15.

Amphotericin B is an antibiotic used particularly for treatment of disease caused by

A disease characterized by persistent painful vesiclesthat occur on the skin or a mucous surface along the distribution of a sensory nerve is

1. 2. 3. 4. 5.

fungi. protozoa. rickettsiae. staphylococci. acid-fast bacteria.

1. 2. 3. 4. 5.

psoriasis. herpes zoster. recurrent herpes. cat-scratch fever. infectious mononucleosis.

16.

22.
The chief complication of nodular prostatic hyperplasia is

Anaerobic bacteria lacking catalase are generally

1. 2. 3. 4.

impotence. hormonal imbalance. urinary obstruction. development of carcinoma.

1. 2. 3. 4. 5.

motile. sensitive to H1O1. insensitive to H11. . able to ferment lactate. producers of 01 and H2O from H201 .

23

23.

The most common type of epithelial metaplasia involves

29.

Histamine release from mast cells requires

1. antibodies (lg) attached to mast cells 2. antigen attached to mast cells and
3.
reacting with antibody. antibodies (lg) attached to mast cells but no reaction with antigen. antigen attached to mast cells but no reaction with antibody.
'

1. regeneration of epithelium in an area


of cutaneous ulceration. 2. replacement of squamous cells by cuboidal cells. 3. replacement of cuboidal cells by columnar cells. 4. replacement of columnar cells by 'stratified squamous epithelium.,

and reacting with antigen.

4.

24.

A benign tumor arising from voluntary muscle is a

30.

1. 2. 3. 4. 5.

leiomyoma. papilloma. rhabdomyoma. leiomyosarcoma. rhabdomyosarcoma.

Serologic and phage typing of pathogenic bacterial species are used to


',0

1. 2. 3. 4. 5.

provide a basis for immunization. determine whether the agent is motile. permit application of specific therapy. establish the genus of the infective agent. identify bacterial strains in disease outbreaks.

25.

Tetanus and diphtheria are similar in nature with respect to 1. bacteremias.

2. aerobic requirements. 3. productiGn of exotoxins. 4. production of endotoxins.


Tertiary syphilis is characterized by presence of

31.

The eclipse stage of a viral infection is characterized by


,

26.

1. 2. 3.
4. 5.

1. a gumma. 2. a chancre. 3. mucous patches. 4. a maculopapular rash. 32.

development of lysogeny. absence of a demonstrable virus particle. a slowly increasing number of virus particles. all of the above. none of the above.

27.

The components in the cell wall of Mycobacteriaceae responsible for acid-fastness are

A common oral manifestation of Addison's disease is

1. 2. 3. 4. 5.

mucoproteins. teichoic aci ds. polysaccharides. lipids and waxes. magnesium ribonucleates.

1. melanosis. 2. xerostomia. 3. glossodynia. 4. "cotton-wool" bone. 5. loss 'of lamina dura.

28.

The spherical body produced under appropriate conditions from certain bacilli by the action of lysozyme or penicillin is.

33.

The most characteristic feature of a neoplasm as opposed to an inflammatory overgrowth is 1. 2. 3. 4. 5. abnormal mitosis. size of the lesion. tendency to grow rapidly. tendency to recur after removal. progressive growth after removal of causative stimuli.

1. 2. 3. .4. 5.

an axostyle. a protoplast. an auxotroph. a prototroph. a chloroplast.

,
34.
Fermentation occurs when the

40.

1. electron donor is an organic compound. 2. final electron acceptor is oxygen. 3. final electron acceptor is an organic
4.

Of the following, specific chemotherapy useful for treatment of

is

s. 35.

compound. final electron acceptor is an inorganic compound. electron donor and the final electron acceptor are inorganic compounds.

l. 2. 3. 4. 5.

measles. smallpox. hepatitis. herpes conjunctivitis. infectious mon<>nucleosis.

41.
With respect to its histologic appearance, biologic behavior and preinvasive states, oral squamous cell carcinoma most closely resem bles

A group of antibiotics related both structurally and by mode of action to the penicillins is

1. 2. 3. 4. S. 36.

breast cancer. cervical cancer. Hodgkin's disease. carcinoma of the lung. carcinoma of the colon.

1. 2. 3. 4.

polymyxins. cycloserines. eeph alosporins. chloramphenicols.

42. The principal source of endemic histoplasmosis infection generally involves

Of the following, the most common site of a basal cell carcinoma is the

1. 2. 3. 4. S. 37.

mosquitoes. food sou rees. dogs and cats. soil and dust. droplet nuclei from human infections.

1. 2. 3. 4. 5.

tongue. gingiva. lower lip. upper face. oral mucosa.

43.
Chest pain associated with lobar pneumonia is attributable to

Increased serum acid phosphatase is clinically significant and aids in the diagnosis'of

1. 2. 3. 4. S.

coexistent pleurisy.

overuse of intercostal muscles. inflammatory changes in alveolar walls. congestion of the pulmonary vascular bed. extension of the inflammation to the bronchi.

1. 2. 3. 4. 5.

Paget's disease of bone. prir'nary hyperparathyroidism. secondary hyperparathyroidism. breast carcinoma with bone metastasis. prostate carcinoma with bone metastasis.

38.

The most active cel/ular structure in controlling the tntake of solutions into bacteria is the 1. 2. 3. 4. capsule. flagella. cell wall. cytoplasmic membrane.

44.

The fading of nuclear chromatin of a necrotic cell is 1. 2. 3. 4. pyknosis. karyolysis. karyorrhexis. hyperchromatism.

39.

Rubeola, pertussis, poliomyelitis and mumps have in commonthe fact that all are

45.

Li~uids destroyed by temperatures over 90 C may be sterilized by

1. 2. 3. 4. S.

vi raf diseases.

vector-borne diseases. characterized by a toxemia. primarily diseases of infants. prevented by active immunization.

1. 23. 4. 5.

dry heat. fiItrati on. autoclaving. centrifugation. fractional sterilization.

25

46.

The poten t toxi ~ called aflatoxin is produced by

52.

l. 2. 3. 4. 5.

A woman has metastatic carcinoma of the jaws. The primary lesion would most likely be found in the 1. 2. 3. 4. lung. colon. breast. cervix.

Mucor. Candida. Aspergillus. Penicillium. Tricophyton.

47.

On the basis of histogenesis and transitions observed in clinical cases, there appears to be a r~lationship between lymphocytic Iympnuma and

53.

An increase in ventricular wall thickness'in paticn ts wi th hypertensive heart disease is an example of 1. 2. 3. 4.

1. 2. 3. 4.

lymphadenitis. lymphoepithelioma. lymphocytic leukemia. infectious lymphocytosis. 54.

neoplasia. metaplasia. hyperplasia. hypertrophy.

48.

The complemen t system may be activated

The renal hormone associated with hypertension is

l. 2. 3. 4. 5.

at (1 and C3. only by way of C1. by interferon during viral infection. QYactivation of any of the nine Components. only when the body temperature exceeds 101.6F. 55.

1. 2. 3. 4. 5.

renin. insulin. angiotensin. parathormone. erythropoietin.

49.

The most common primary malignant neoplasm of the lung is 1. 2.


.. 3.-

Cells, tissues or organs that are transferred from a donor to a genetically different recipient of the same species are

adenocarcinoma. bronchial adenoma.


al\leolarcell carcinoma.

1. isografts. 2. allografts. 3. he terogra fts. 4. au toch th on ou s. 5. None of the above


56. 50.
Unlike OPT vaccine, BCG vaccine consists of

4. 5.

squamous cell carcinoma. undifferentiated carcinoma.

A candidate virus for the induction of cervical carcinoma is

1. 2. 3. 4. 5. 51.

killed virus. avirulent virus. killed bacteria. avirulen t bacL~ la. a mixture of toxoids.

1. 2. 3. 4. 5.

adenovirus. C-type virus. varicella zoster. Epstein-Barr virus. herpes virus hominis Type 2.

Of the various histologic patterns of Hodgkin's disease, the type which offers the most favorable prognosis is characterized by

57.

In most slow viruses, tissue damage occurs in the

l. 2. 3. 4.

lymphocytic predominance. a mixed cellular infiltrate. reticular fiber predominance. numerous Reed,Sternbergcells:

1. 2. 3. 4. 5.

lung. heart. brain. spleen. kidney.

&

58.

The most common causative organism in gram-negative sepsis is

64.

The histologic pattern in a renal ischemic infarct is 1. 2. 3. 4. 5.


caseous necrosis. gummatous necrosis. gangrenous necrosIS. coagulation necrosis. liquefaction necrosis.

1. 2. 3. 4. 5.

pneumococcus. . . virus organism. Escherichia coli. Streptococcus viridans. Streptococcus pyogenes.

65.

59.

Fluoride is most effective and safe as a


prophylactic measure when it is

The initial bacteria observed in dental plaque are 1. 2. 3. 4.


spirochetes. streptococci. staphylococci. actinomycetes.

1. 2. 3. 4. 5.

in the F2 state. applied topically. taken in travenousl y. supplied in sucrosc. added to the water supply.

66.

In the normal healthy mouth, the microflora tends to be mainly

60.

The most common etiologic agent of septic arthritis in adul ts is

1. 2. 3. 4.

1. 2. 3. 4. 67.

anaerobic, aerobic and proteolytic. anaerobic, aerobic and facul tative. anaerobic, facultative and acidogenic. none of the above.

Neisseria gonorrhoeae. Hemophilus influenzae. Streptococcus pyogenes. Streptococcus pneumoniae.

Dextrans are extracellular polysaccharides readily produced by . 1. 2. 3. 4.


Streptococcus Streptococcus Streptococcus Streptococcus mitis. mutans. pyogenes. sa/ivarius;

61.

The delayed type of hypersensitivity trans ferred by

can be

1. 2. 3. 4. 5.

lysozyme. antibodies. mast cells. plasma cells. sensitized lymphocytes.

68.

A bacterial muration leading to the requirement for a single amino acid is due to

62.

l. 2. 3. 4. 5.
69.

lack of m RNA. loss of ability to utilize glucose. absence of a single enzyme activity. absence of cell wall polysaccharide. absence of several enzyme activities.

The basic effect of x-radiation upon living tissues is

1.
2. 3. 4. 5:

. ionization.

denaturation. agglutination. precipitation. cauterization.

Transmission of the hepatitis B virus may be through (a) (b) (c)


parental administration of blood or blood products. use of con laminated instrumen ts or injection equipment. microabrasions in skin and mucous membranes by contact with in fected materials (blood, saliva, feces).

63.

The principal antibody-producing cell is the

1. 2. 3. 4. 5.

mast cell. Iymphocyte. macrophage. eosinophil. plasma cell.

1. 2. 3.

(a) only (a) and (b) (a) and (c)


.

4.
5.

(b) and (c) All of the above

27

70.

Rickettsiae are similar to bacteria in that they (a) (b) (c) (d) (e) multiply by binary fission. can be cultured on enriched blood agar. are capable of producing heat-resistant endospores. possess an energy-yie! ding, autonomous enzyme metabolism. are susceptible to the lethal effects of certain antibiotics.

73.

Arrange the following types of cells or tissues in decreasing order of radiosensitivity from most sensitive to least sensitive. (a) (b) (c) "(d) (e) Osteocyte Endothelium Smooth muscle Spermatogon ium Intestinal mucosa

1. 2. 3. 4. 5. 6.

(a), (b) and (c) (a), (c) and (d) (a), (d) and (e) (b), (c) and (e) (b), (d) and (e) All of the above

1. 2. 3. 4. 5.

b, c, a, e, d c, a, b, d, e d, b, c, a, e d, e, b, c, a e, d, c, a, b

74.

Cells of which of the following retain a laten t capacity for mitotic division?

"

71.

Significant functions of polymorphonuclear leukocytes in inflammation are (a ) (b) (C) (d)


.

(a) (b) (c) (d) (e)

Liver Bone marrow Cardiac muscle Salivary glands Neurons

"

replication of new cells. phagocytosis of bacteria. elaboration of proteolytic enzymes. elaboration of an tibodies.

1. 2. 3. 4. 5.

(a), (b) and (c) (a), (b) and (d) (a), (c) and (d) (b), (c) and (e) (b), (d) and (e)

1. (a) and (b) 2. (a) and (c) 3. (b) and (c) 4. (b) and (d) 5. .. (c) and (d)

75.

Which of the following statements explain the advantageous effects of fibrin formation?

(a) (b)

72.

Neonatally thymectomized mice and nude mice exhibit (a) ~b) (c) reduced numbers of T.lymphocytes. inability to reject allografts. reduced antibody production to most antigens.

(c) (d)

Fibrin transforms into fibroblasts serving in repair. Fibrin generates hyaluronidase to dissolve exudates. Fibrin serves as scaffolding for fibroblasts to proliferate in repair. Fibrin serves to wall off attacking agents to prevent further spread.

1. 2. 3. 4.

(a) and (b) (a) and (c) (b) and (c) All of the above

1. a) and b) 2. a) and c) 3. ! a) and d) 4. (b) and (c) 5. (b) and (d) 6. (c) and (d)
~

76.

Which of the following are possible sequelae to acute appendicitis? (a) (b) (c) (d) Generalized peritonitis Periappendiceal abscess formation Pylephlebitis Hepatic abscess

80.

When horse serum is injected intravenously into a rabbit and again into the skin two or three weeks later, what is the necrotizing reaction that occurs at the site of the second injection?

1. (a), (b) and (c) 2. (a), (b) and (d) 3. (a), (c) and (d) 4. (b), (c) and (d) 5. All of the above 81. 77.
Which of the following are related to streptococcal cross-antigeniCity?

1. 2. 3. 4. 5.

Atopy Anaphylaxis Serum sickness Arthus phenomenon Prausnitz-Kustner reaction

,. i I

I i I

Recurring attacks of bronchial asthma may predispose to which of the following diseases?

(a) (b) (c) (d) (e) 1. 2. 3. 4. 5. 6.

Sympathetic opthalmia Milroy's disease Systemic lupus erythematosus Rheumatic fever Acute glomerulonephritis

1. 2. 3. 4.

Empyema Emphysema Tuberculosis Cor pulmonale

82.

(a) and (c) (a) and (d) (b) and (c) (b) and (e) (c) and (e) (d) and (e)

Which of the following is classified as a hemolytic anemia? 1. 2. 3. 4. Aplastic anemia Pernicious anemia Sickle-cell anemia Iron deficiency anemia

83. 78.
Which of the Jollowing is the best method to avoid hepatitis B cross infections among dental patien ts?

Which of the following would be expected to occur when T-Iymphocytes from a patient with chronic periodontitis are cultured in vitro with dental plaque an tigen?

1. - 2. 3.

4. 5.

Soak instrumen ts in a phenolic disinfectant for 20 minutes. Use a two-minute surgical scrub for hands and all items soiled by blood. Autoclave or dry heat sterilize all instruments used in treatment. Boil all needles and cu tting instrumen ts for five minutes in water containing anti-rust chemicals. Autoclave or dry heat sterilize those instruments used to treat patients who give histories of the disease.

1. Release of histamine 2. Produc tion of speci fic IgG 3. Release of slow-reacting substance (SRS) 4. Production of macrophage inhibition
factor (MIF)
.'

84.

Which of the following sequelae of fatty nutritional cirrhosis is most likely to result in death?

1. 2. 3. 4.

Ascites Jaundice Hepatomegaly Esophageal varices

79.

Which of the following hormones has the greatest effect on granulation tissue in healing wounds?

85.

Which of the following groups of microorganisms is the most important causative agent of dental caries?

1. 2. 3. 4. 5.

Thyroxin Estrogen Cortisone

Parathormone

'

Antidiuretic hormone

1. Aciduric 2. Acidogenic 3. Basophilic 4. Proteolytic 5. Gram-negative rods

29

86.

Which of the following is a feature of infectious mononucleosis?

92.

Each of the following is commonly associated with congestive heart failure EXCEPT

1. It frequently occurs in aged patients. 2. Peripheral blood shows marked eosinophilia. 3. . The mortality rate is approximately 4.
5. 20 per cent.
.

The serum o(the patient will agglutinate sheep red cells. There is a marked plasma cell infiltra-te in to the attached gingiva.

1. 2. 3. 4. 5.

dyspnea. cyanosis. anasarca. ankle edema. passive congestion of the liver.


. .

87.

Which of the following relates to lysogeny?

93.

Osteitis deformans is characterized by each of the following EXCEPT

1. The lytic cycle occurs after curing. 2. Curing represents loss of repressor. 3. Prophage is integrated into bacterial 4.
chromosome. None of the above

1. reversal lines with a characteristic, 2. decreased serum calcium and elevated 3. predisposition to development of
.

mosaic pattern.

serum phosphorus.

4. irregular resorption and replacement 88.


Whichof the following is recommended for disinfecting dental units and handpieces? of bone with a poorly mineralized osteoid matrix.

osteogenic sarcoma.

1. 2. 3. 4.

Clorox@ and Lysol@ (2%)

Gluteraldehyde (Cidex@)

Betadine and zepharin (1: 750) 2% staphene or benzalkonium chloride in 70% alcohol

94.

Of the following, the tissue with the least ability to regenerate is

89.

Which of the following species is found consistently in saliva and on oral soft tissues?

1. 2. 3. 4" 90.

Streptococcus Streptococcus Streptococcus Streptococcus

mutans sanguis pyogenes salivarius

1. 2. 3. 4. 5.

bone. liver. neuron. collagen fiber. . squamous epithelium.

Each of the following is a histologic feature of malignant growth EXCEPT

95.

The form of bacterial gene transfer that is least susceptible to DNAase and does NOT require cell-to-cell contact is

1. 2. 3. 4. 5.

aplasia. anaplasia. pleomorphism. hyperchromatism. abnormal mitosis.

1. 2. 3. 4.

transition. conjugation. transduction. transformation.

91.

Jaundice is usually a result of each of the following EXCEPT

96.

Which of the following is NOT characteristic of plasmids?

1. 2. 3. 4.

s.

hepatitis A. hemolytic anemia. vitamin K deficiency. carcinoma of the common bile duct.

gallstones.

1. 2. 3. 4.

Confer conjugal fertility Carry genetic information Exist as circular RNA molecules Exist as extrachromosomal elements in bacteria

30

..

97.

Which of the following is NOT characteristic of carcinoma in situ?

99.

Whichof the following is NOT a complication of untreated left-sided heart failure?

1. Pleomorphism 2. Disorderly maturation 3. Hyperchromatic nuclei 4. Disruption of the basement membrane 98.

1. 2. 3. 4. 5.

Orthopnea Pleural effusion Bacterial pneumonia Myocardial hyperplasia Paroxysmal nocturnal dyspnea

Which of the following is NOT a feature of infectious mononucleosis? 100.

1. 2. 3. 4. 5.

Splenomegaly Necrotizing pharyngitis Depressed heterophile titer Abnormal lymphocytes in peripheral blood Marked increase in the number of circulating lymphocytes

Whichof the following is NOT associated with urinary outflow obstruction?

1. 2. 3. 4. 5.

Cystitis Urolithiasis Hydronephrosis Pyelonephritis Benign prostatic hypertrophy

..

NATIONAL BOARD DENTAL EXAMINATIon PART I ANSWER KEY MICROBIOLOGY/PATHOLOGY DECEMBER 1979

No. -

Ans. 1 4 4 1 1 2 5 2 3 3 1 2 1 5 1 3 4 1 2 1 2 2 4 3 3

No. Ans. 26. 27. 28. 29. 30. 31. 32. 33. 34. 35. 36. 37. 38. 39. 40. 41. 42. 43. 44. 45. 46. 47. 48. 49. 50. 1 4 2 1 5 2 1 5 3 2 4 1 4 5 4 3 4 5 2 2 3 3 1 2 4

No. 51. 52. 53. 54. 55. 56. 57. 58. 59. 60. 61. 62. 63. 64. 65. 66. 67. 68. 69. 70. 71. 72. 73. 74. 75.

Ans. 1 3 4 1 4 5 3 3
.5

No. 76. 77. 78. 79. 80. 81. 82. 83.


84.

Ans. 5 6 3 3 4 2 3 4 4 2 4 3 2 4 1 4 3 2 3 3 3 4 3 4 1

1.
\

2.

I.

3.
4.

5. 6. 7.
8. 9. 10. 11. 12. 13. 14. 15. 16. 17. 18. 19. 20. 21. 22. 23. 24. 25.

1 5 1 5 4 2 3 2 3 5 3 3 4 4 2 6

85. 86. 87. 88. 89. 90. 91. 92. 93. 94. 95. 96. 97. 98. 99. 100.

DENTAL ANATOMY

1.

In the drawing below, the arrows represent possible contacting areas for which of the following maxillary cusps?

5.

In general, how do the eruption schedules of permanent mandibular teeth and permanent maxillary teeth compare? 1. 2. 3.
Mandibular teeth erupt earlier. Maxillary teeth erupt earlier. Mandibular and maxillary teeth erupt simultaneously.

1. Facial cusps of premolars 2. Lingual cusps of premolars 3. Facial cusp of second premolar and mesiofacial cusp of first molar 4. Lingual cusp of second premolar and
mesiolingual cusp of first molar

6.

The permanent maxillary canine opposes which of the following mandibular teeth? 1. Canine and first premolar 2. lateral incisor and canine 3. First premolar only 4., Canine only

2.

Identify the mandibular movement in the drawing above.

7.

1. Protrusive movemen t 2. Right lateral (working side) movement 3. Right lateral (non-working side) movement 4. left lateral (working side) movement 5. left lateral. (non-working side) movement

Erupted teeth of maxillary and mandibular !'efthalves of a mixed den titian are shaded below. Approximately how old is the child?

3.

In the drawing below, the arrow represents a possible contacting area for which of the following mandibular cusps?

1~~&~~Jm~ . !~Aj*i
1~~1 ""

I',~~~"
1. 2. 3. 4. 5.
Cusp of canine Facialcusp of first premolar lingual cusp of first premolar Facial cusp of second premolar Lingual cusp of second premolar

~~

1. 2. 3. 4. 5. 6.

2 years 5 years 6 years 8 years 11years 12 years

4.

Identify the mandibular movement in the drawing above.

8.

1. 2. 3. 4. 5.

Protrusive movement Right lateral (working side) movement Right lateral (non-working side) movement left lateral (working side) movement left lateral (non-working side) movement

Which of the following premolars generally poses the greatest problem when root canal therapy, extraction or apicoectomy is being considered?

1. 2. 3. 4.

Mandibular first Mandibular second Maxillary first Maxillary second

9.

Which primary tooth has a facial cervical - ridge which is so distinctly prominent that it is uniquely different from other teeth?

14.

Which of the following primary molars bears the greatest resemblance to a premolar? 1. 2. 3. 4. Maxillary first Maxillary second Mandibular first Mandibular second

1. 2. 3. 4. 5.

Mandibular canine Mandibular first molar Maxillary central incisor Maxillary canine Maxillary second molar

15.
10. Ordinarily, a 6-year-old child would have which teeth clinically visible?

Which of the following permanen t teeth has a mesiodistal measurement greater lingually than facially? . .
1.

I
I

1. All primary teeth only 2. All primary teeth and permanent first 3. 4.

I I I

5.

molars Primary posterior teeth, permanent first molars and permanent central incisors Primary canines, permanent first molars and permanent mandibular incisors Primary posterior teeth and permanent central and lateral incisors

2. 3. 4. 5.

Maxillary canine Maxillary first molar Mandibular first molar Maxillary first premolar Mandibular first premolar

16.

The transverse groove of the oblique ridge of a permanent maxillary molar connects which of the following pits?

11.

Which of the following teeth usually has the steepest cusp inclines?

1. 2. 3. 4.

Maxillary first premolar Mandibular second premolar Permanent maxillary first molar Permanent mandibular first molar

1. 2. 3. 4. 5. 6.

Distal and distofacial Central and distofacial Central and facial Central and distal Central and mesial Central and lingual

17.

12.

The anatomy of the mesial surface of which of the following teeth is most likely to cause difficulty in restorative dentistry?

From a proximal view, which of the following permanent teeth tends to be positioned in the arch with its axis most nearly vertical?

1. 2. 3. 4.

Mandibular first molar Maxillary first premolar Maxillary central incisor Maxillary second premolar

1. 2. 3. 4. 5.

Maxillary canine Maxillary lateral incisor Maxillary central incisor Mandibular lateral incisor Mandibular central incisor

13.

Deep concavities occur on which of the following permanent maxillary tooth surfaces?

18.

1. Mesials of central incisors and mesials 2. 3. 4. 5.


of first premolars Mesials of canines and mesials of first molars Distals of second premolars and distals of second molars Mesials of first premolars and distals

When teeth are in normal alignment and viewed from the occlusal, which teeth are so placed in the arch that a straight line can be drawn through their contact areas?

1. Maxillary anteriors 2. Mandibular anteriors 3. Maxillary second premolars and all


4. 5. 6. maxillary molars Mandibular second premolars and aU mandibular molars Maxillary molars only Mandibular molars only

of first molars

Disuls of first molars and distals of second molars

34

19.

In which of the following roots would a divided pulp canal be most likely to occur? 1. 2. 3. 4. 5.
Maxillary central incisor. Maxillary canine Mandibular canine Lingual root of maxillary first molar Distofacial root of maxillary second molar

24.

In normal occlusion, when the mandible makes a right working movement, which of the following may be in relation to the mesiolingual cusp of the maxillary left first molar? Sulcus betwe'en distofacial and distal cusps of the mandibular first molar 2. Sulcus between lingual cusps of the mandibular first molar 3. Embrasure between mandibular first molar and second premolar 4. Embrasure between mandibular first and 5ecol"ld molars '5. None of the above

20.

Approximately what percentage of root calcification is completed at the time the tooth erupts?

1. 2. 3. 4. 5. 6.

20 30 50 75 90 100

2:=; .

Pictured below is the au tline of a cross section made at the cervix of which of the following permanent teeth?

21.

Which of the permanent incisors is most frequently markedly concave on the lingual surface?

1. Maxillarycen tral incisor 2. Maxillary lateral incisor 3. Mandibular lateral incisor 4. Mandibular central incisor

22.

Which of the following features would be of greatest value in distinguishing between a primary mandibular second molar and a permanent mandibular first molar?

1. 2. 3. 4.

Maxillarycanine Maxillary lateral in~isor Mandibular central incisor Mandibular second premolar

1. 2. 3. 4.

26.
Number of roots Number of cusps Comparative size of distal cusps Arrangement of grooves on occlusal surfaces

In the permanent dentition, pulp chamber morphology makes which of the following pulp horns most subject to accidental exposure in the preparation of a ClassII cavity?

23.

Which of the following statements'is applicable to both the maxillary first premolar and the mandibular second premolar?

1. 2. 3. 4. 5.

Mesiolingualofa maxillary first molar Mesiofacialof a mandibular first molar Distofacial of a mandibular first molar Facial of a mandibular first premolar Lingual of a mandibular second premolar

1. One root is present. 2. The number of cusps is relatively 3. 4. 5. 6. 7.

constant. The crown is concave on its mesial surface. The root is extremely concave on the mesial aspect. The lingual cusp is lower in height than the facial cusp. Occlusal anatomy may exhibit a high degree of variation. There may be a groove on the lingual surface of the crown.

27.

Whenextracting a primary maxillary incisor in which the root has been partially resorbed because of pressure from its developing succedaneous tooth, which aspect of the remaining primary root will usually be longest and attached most securely to the gingiva?

1. 2. 3. 4.

Distal Facial Mesial Lingual

35

28.

What would happen if the mandible were forced into centric relation from the rest position?

33.

Which of the following sketches illustrates the facial view of the permanent mandibular right canine?

l. 2.

3. 4.
29.

The patient's reflex neuromuscular defense would resist the applied force. The mandible would be positioned in centric relation. The neuromuscular system would position the condyles. None of the above. The mandible cannot be retruded from the rest position.

The widest incisal or occlusal embrasure is normally found between which of the following permanen t maxillary teeth?

1. Central incisors. 2. Central and lateral incisors 3. Lateral incisor and canine 4. Canine and first premolar. 5. Arst and second premolars 30.
Which sulcus of the mandibular first molar does the maxillary mesiolingual cusp pass through in a lateral excursive movement on the working side?

34.

1. 2. 3.. 4. 31.

Facial Lingual Central Distofacial

Which of the following sketches illustrates the occlusal view of the mandibular right first premolar?

In carving an occlusal amalgam restoration in a permanen t mandibular second molar, which of the following shapes is the most typical groove pattern?

~
5

I.--"'--, . ,, , ,

1. 2. 3. 4. 5.

H L U Y + 35. Whichof the following sketches illustrates the mesial view of the permanent mandibular left lateral incisor?

32.

Which of the following sketches illustrates the occlusal view of the permanent mandibular right first molar?

36.

Which of the following sketches illustrates the occlusal view of the primary maxillary right second molar?

40.

When waxing a hypothetical ideal occlusion, which of the following factors must be incorporated into the design?

(a)
(b)

tj
1

U'C!i'1

".. ...' "<f'.-'


2

@
r

. /.. I.

. ,. . .-~ 5

(c)

(d)

Guiding (non-supporting) cusps are related to interproximal areas or developmental grooves. Supporting cusps are related to marginal ridges and fossae. Supporting cusps are aligned with supporting cusps of the same quadran t. Guiding (non-supporting) cusps overlap facial to mandibular teeth and lingual to maxillary teeth,

37.

A maxillary first premolar has how many pulp horns?

1. (a) only 2. (a), (b) and (c) 3. a), (c) and (d) 4. b) ani y 5. c) only 6. .c) and (d) only 7. All of the above

38.

41.
Which of the following statements are applicable to the maxillary second premolar?

In a normal 6cclusio~, the mandibular central incisors will contact the maxillary incisors in which of the following movements?

(a) (b) (c) (d) (e) (f)

Usually one root is present. The cusps may have vastly differen t forms. The root is extremely concave on the mesial aspect. The crown is extremely concave on the mesial surface. There may be a groove on the mesiolingual aspect of the crown. Facial and lingual cusps are approximately the same size.

(a) (b) (c) (d)

1. 2.
3; 4. 5. 6.

Protrusive lateral (working) lateral (non-working) lateral protrusive (a) and (b) only a), (b) and (d) a) and (d) only b) and (c) (c) only (d) only
.

1. (a ' c) and (f) 2. a, d) and (f) 3. a, ! e) and (f) 4. a and (f) only 5. b), (d) and (e)

42.

Which of the following permanent teeth sometimes have three cusps and a singie Cen tral pit? (a) b) c) d) e) Mandibular second premolar Mandibular second molar Maxillary second premolar Maxillary second molar Maxillary third molar

39.

Which of the above statements are applicable to the mandibular first premolar?

l. 2. 3. 4.

a ' b) and (e) a, c) and (e) a, ! .! e) and (f) b), (c) and (f)

1. 2. 3. 4.

(a), (c) and (e) . (d) and (e) (a), (b) and (c) . (b) and (d)

37

43.

Whichof the following are characteristic of a permanent maxillary central incisor but not of a permanent maxillary 14teralincisor? The greatest mesiodistal measurement is more than the greatest faciolingual measurement. (b) . It has a shallow, broad lingual fossa on the incisal one.half of the lingual surface. (c) The lingual fossa is bounded by the mesial marginal ridge, the distal marginal ridge, the incisal ridge and the cingulum. (d) The cervical line curves incisally more on its mesial surface than on its distal surface. (e) The cervical outline of the facial surface is a broad curve that is sometimes described as part ora semicircle. (a)

45.

In comparing primary and permanent dentitions, which of the following are notable differences?

(a) (b)
(c) (d) (e)
.

(f)

Cervical ridges of permanent teeth are more pronounced. Crowns of primary incisors are shorter compared with mesiodistal diameters. Root trunks of primary teeth are less pronounced. Enamel is thinner in primary teeth com pared with buIk 0f the crowns. Pulp cavities are larger in permanent teeth. Roots of permanent teeth are longer and slimmer.

1. a), (b) and (c) 2. a), (c) and (e) 3. l b) and (c) only 4. (b) and (e) S. (c) and (d) 6. (c) and (e) only

1. a), (b) ahd (c) 2. a), (e) and (f) 3. b), ,(c) and (d) 4. b), (c) and (e) 5. (c), (d) and (e) 40.

Loss of proximal contact could be expected to result in a) b)


! ~~

(e)
.

drifting of teeth. food impaction. bone loss. damage to the interdental papilla. unfavorable occlusion.

44.

Whichof the following features of the dentition directly affect the health of the periodontal ligament and its hard tissue anchorage in terms of resisting occlusal force?

1. a) (b), (c) and (d) 2. a), (c) and (e) 3. . b), (c) and (e) 4. j d) and (e) S. All of the above'
47. This outline of a pulp cavity indicates that this anterior tooth probably

(a) Anterior teeth have slight or no (b)


(c) (d) (e) (f) contact in the intercuspal position. The occlusal table is less than 60 per cent of the overall faciolingual width of the tooth. Thjrd molars are usually out of occlusion. Maxillary incisors are wider mesiodistally than faciolingually. The occlusal table of the tooth is generaJly at right angles to the long axis of the tooth. Crowns of mandibular molar are incfjned about 20 toward the lingual. d) f) e) 2 f)

a) b) c) d)

1. 2. 3. 4. 5. 6.

has three well-developed lobes. experienced trauma. is that of an old person. has a triangular shape in cervical cross section.

(a, b), (c) and (a, b), (e) and a, C ' (d) and a, c, (d) and b), (cI , (e) and ! b), (d), (e) and

1. 2. 3. 4. 5.

a) and (b a) and (d b) and (c b) and (d\ c) and (d)

. ! ! i i I

" 48.
Permanent mandibular molars are distinguished from permanent maxillary molars in that permanent mandibular molars have
(a) (b) two roots, one mesial and one distal. crowns that are broader. faciolinguaJly than mesiodistally. lingual cusps almost equal in sizc. crown axes inclined toward the facial.

51.

In the intercuspal position, the maxillary posterior teeth oppose


a) b) c) l d) facial inclines of facial cusps. lingual inclines of facial cusps. facial i11c1inesof IinguaJ cusps. lingual inclines of lingual cusps.

(c) (d)

1. 2. 3. 4. S.

a) and (bt a) and (c) b) only b),(c) and (d) c) and (d) only

1. a ' (b) and (c) 2. a , (b) and (d) 3. a i and (c) only 4. b) and (c) only S. All of the above

52.

In comparison with the mandibular canine, the maxillary canine


a) b) c) d)

49.

Areas of teeth most susceptible to dental decay are

(e)

(a)
. (b) (c) (d) (e)

cusp tips. pits. facial surfaces cervical to the height of contour. facial surfaces occlusal to the height ofcontour. proximal surfaces. (c) (e) (e) (e)

has a relatively shorter crown. is shorter. has a more pronounced cingulum. has a less pronounced developmental depression mesially on the root. has a cusp tip more nearly centered over the root, viewed from the facial aspect.

1. 2 3. 4. S. 53.

(ai (c) and (d) ,a" (c) and (e) (b) and (e) (c) only All of the above

1. 2. 3. 4. S.

a), (b) and b), (c) and b), (d) and c), (d) and e) only

A Class III malocclusion is very often found when

1- mandibular and maxillary arches are proportionate to each other in size. 2. the maxillary arch is larger than the
3.

mandibular arch. the mandibular arch is larger than the maxillary arch.

54. 50.
The primary cusp triangle on the occlusal surface of a permanent maxillary molar is formed by the
.

When the incisal edges of anterior teeth are placed in end-to-end contact, the condyles of the mandible move

.b)
c) d) e)

a)

distolingual cusp. mesiolingual cusp. cusp of Carabelli. mesiofacial cusp. distofacial cusp.

1. 2. 3. 4.

backward. forward only. downward only. downward and forward.

1. a), (b and 2. a), (b and 3. a), (d j and 4. .b), (d) and S. (c), (d) and

55.
(c) (e) (e) (e) (e)

The ~nterdental papilla is located in the

12. 3. 4.

facial embrasure. lingual embrasure. cervical embrasure. occlusal embrasure.

39

;~

56.

In a normal dentition in the- intercuspal position, the mesiolingual cusp of the permanent maxillary first molar opposes the mandibular first molar in the

62.

At the same time as the suprahyoid muscles contract during swallowing, a downward displacement of the mandible is prevented by con trac tion of the

1. cen tral fossa. 2. distal fossa. 3. mesi al fossa.


57.

1-

2. 3. 4. 5.

infrahyoidmuscles. .
masticatory muscles. anterior cervical muscles. posterior cervical muscles. muscles of facial expression.

Viewed occlusally, the lingual portion of the crown outline is generally greater than the facial portion of the crown outline for the permanent

63.

1. 2. 3. 4.
58.

maxillary first molar.. mandibular first molar. maxillary second molar. mandibular second molar.

The primary teeth that present the most outstanding morphologic deviations from permanent teeth are the

The glenoid fossa is an oval cavity or a depression in the 12.

2. 3. 4. 5.
64.

1-

central incisors. lateral incisors. canines. first molars. second molars.

3.

maxilla, just anterior to the malar process. temporal bone, just anterior to the auditory canal. mandible, just posterior to the

In an ideal centric occlusion, the mesiofaciaJ cusp of the permanent maxillary first molar opposes the

internalobliqueridge.

1. central fossa of the mandibular first 2. mesial fossa of the mandibular first
molar. molar. facial embrasure between mandibular second premolar and first molar. sulcus of the distofacial groove of the mandibular first molar. sulcus of the mesiofacial groove of the mandibular first molar.

59.

When in its proper position relative to the plane of occlusion, the crown of the mandibular second molar inclines

3. 4. 5.

1. 2. 3. 4.
60.

distally and facially. mesially and facially. mesially and lingually. distally and lingually.

When two proximal surfaces diverge from an area of contact, an embrasure is formed 1- lingually only. 2. lingually and facially only. 3. lingually and occ/usally only.
I

65.

Lingual cusps of mandibular molars oppose

4. Iinguall Y facially, oee!usalJy and


cervically.

1. 2.. 3. 4. 5.

grooves only. marginal ridges only. grooves and embrasures only. marginal ridges and embrasures only. grooves, embrasures and marginal ridges.

61.

As the teeth are closed into the intercuspal position, the final movement of the mandible is directed by the

66.

The free gingivamay be described as

1. 2. 3. 4. 5.

height of the free way space. strength of the muscles of mastication. cusp-fossa relationship of opposing teeth. ligaments that extend from the skull to the mandible. shape of the articular eminence of the temporomandibular joint.

1. filling the interproximal area. 2. forming the wall of the gingivalsulcus. 3. marking the opening of the gingival 4.
5.

sulcus. being attached to the cementum and the alveolar bone. extending from the free marginal groove to the alveolar mucosa.

~ ~
67.
The masticatory movemen t of the mandible may be described as

73.

1. vertical and tear.drop in appearance. 2. occurring on the non-working side. 3. the same as movements in lateral 4.
contacting movement. resembling the movements in Gothic arch tracing.

Morphologically, the permanent mandibular lateral incisor is almost identical to the permanent mandibular central incisor. The difference is that

1. the permanent mandibular lateral incisor


2. is usually slightly larger. the permanent mandibular central incisor has an elongation of the distoincisal angle distolingually. when the permanent mandibular central incisor is viewed incisally the crown appears to be slightly rotated on its base.
I

3.

68.

A maxillary right canine may be distinguished from a maxillary left canine because

1. the root cu rYesto the mesial in the apicalone-third. . 2. the distal half of the canine shows 3. facially, the cusp tip is placed distal 4.
more convexity than the mesial half. to a line which bisects the crown and the root. lingually, the cervical line slopes mesially.

74.

The lingual cusp of the maxillary first premolar is inclined

1. 2. 3. 4. 5.

distally. mesially. distolirigually. mesiofacially. directly over the center mesiodisully.

69.

The maxillary canine.erupts

l. after lateral incisor and first premolar. 2. before lateral incisor and first premolar. 3. after first premolar bu t prior to lateral 4.
incisor. after lateral incisor but prior to first premolar.

75.

The premolar that most frequently has a single central pit: is the

1. 2. 3. 4.

maxillary first. maxillary second. mandibular first. mandibular second.

70.

Spacing between anterior teeth in the primary dentition is most fr~quently caused by

76.

1. 2. 3. 4.

thumb-sucking. tongue thrusting. growth of the dental arches. pressure from succedaneous teeth.

In idea1permanent tooth relationship, the tip of the mandibular canine in lateral excursion passes

1. distal to the tip of the maxillary canine 2. mesial to the tip of the maxillary canine
cusp. cusp.

71.

The small ridges, perikymata, seen on facial surfaces of canines are the result of

3. directly in line with the maxillary canine 4. through the embrasure between maxillary
canine and first premolar. cusp tip.

1. 2. 3. 4. 5. 72.

hyperplasia. hypocalcification. normal development. interstitial development. early ameloblastic degeneration.

77.

In the triangle formed by the projection of the orifices of the canals of a maxillary first molar, the

I I

The permanent mandibular canine root is sometimes bifurcated into

1. angle at the distolingual canal is obtuse. 2. angle at the mesiofacial canal is obtuse. 3. line connecting mesial with distal is
4. longest line connecting mesial with lingual is longest. 5. line connecting distal with lingual is longest.

1. facial and lingual roots. 2. mesial and distal roots. 3. neither ortheabove. The mandibular
canine root does not bifurcate.

41

78.

The height of contour occlusocervically is located within the mi.ddlethird of the 1. 2. 3. facial surface of a permanent mandibular first molar. lingualsurface of a permanent maxillary first molar. distal surface of a permanent central inCisor. . mesialsurface of a permanent canine.

84.

In right lateral (working) movement, the mandibular right central incisor usually contacts the maxillary

4.

l. 2. 3. 4. 5.

right central incisor only. righ t and left central incisors. right centra! and lateral incisors. right lateral incisor only. left lateral incisor only.

79.

85.
Radiographically, the anterior p<)latine foramen is roughly a

The deflective function of mesiofacial and distofacialline angles protects

1. 2. 3. 4. 5.

triangular radiolueent area. elliptical radiolucent area. elliptical radiopaque area. circular radiolucent area. circular radiopaque area.

1. 2. 3. 4. 5.

proximal gingiva. the faCialpart of the interdental papilla. the middle part of the interdental papilla. the lingual part of the interdental papilla. a/l of the above.

80.

86.
In the diagram of Posselt's envelope of motion, the maximum intercuspal position is the

The incisal edges and the incisal thirds of facial surfaces of mandibular incisors and canines generally oppose lingual surfaces of maxillary incisors and canines

l. 2. 3. 4.
81.

lowest point. most anterior point. most posterior point. most superior point.

1. within the incisal thirds. 2. at the linguocervical ridges. 3. immediately incisal to the cingula
4.
on the lingual ridges. at the junctions of middle and cervical

thirds.

Under normal conditions, the presence of mamelons in a 14-year-old patientis indicative of

5. immediately gingival to the junctions of


middle and incisal thirds.

1. 2. 3. 4. 5. 82.

f1 uorosis. malnutrition. malformation. malocclusion. enamel composition.

87.

The periodontal ligament in the region of the permanent mandibular canine tends to become thinner with age, probably as the result of

I
t
I

When viewed lingually, the permanent maxillary first molar has its lingual root apex in line with the

1. 2. 3. 4.

1. 2. 3. 4. 5. 83.

cementum and bone deposition. downgrowths of gingivalepithelium. cell rests forming gingivalcysts. horizontal and oblique fibers reversing function with age.

distolingual groove. distolingualline angle. distolingual cuspal eminence. heigh t of the mesiolingual cusp. mesiodistal diameter midpoint.

I ~
88.
( In the intercuspaJ position, the lingual cusp of the maxillary second premolar
con tac ts the
p

I
!

f
i , I ! 1 . !

The greatest faeiolingual diameter of 3. permanent mandibular second molar is located in the

1. distal fossa of the mandibular second


2. 3. 4. premolar. mesial fossa of the mandibular first molar. distal marginal ridge of the mandibular second molar. distal marginal ridge of the mandibular first premolar.

1. 2. 3. 4.

distal third of the crown. mesiaJthird of the crown. middle third of the crown. None of the above

.
89.
Opening the mouth maximally from retruded position causes the mandible to 1. 2. 3. 4. 90.
translate. rotate. rotate, then transla tee translate, then rotate.

~ ~

95.

Developmentally, the three major cusps that form the primary cusp triangle on a maxillary molar tooth are

1. 2. 3. 4.

mesiofacial, distofacial and mesiolingual. mesiofacial, distolingual and mesiolingual. mesiofacial, distofacial and distolingual. mesiofacial, distofacial and cusp of Carabelli.

In comparing permanent and primary canines, the newly erupted primary maxillary canine has

1. 2. 3. 4. S.

a sharper, more pointed cusp tip. a blunter, more flattened cusp tip. a cingulum~ that occupies a greater portion of the lingual surface. a shorter root, compared to crown length. no lingual ridge between the cusp tip and the cingulum.

96.

Viewed from the occlusal, the basic coronal outline of the mandibular second premolar is

1. 2. 3. 4.

rectangular. rhomboidal. triangular. square.

91.

Maxillary facial and mandibular lingual cusps require sufficient occlusal length and horizontal overlap for

97.

1. 2. 3. 4. S.
92.

Each permanent tooth has a greater dia~ter faciolingually than mesiodistally with the exceptions of the (a) (t ~ lcj (d) (e) mandibular first molar. maxillary first molar. maxillary central incisor. mandibular lateral incisor. mandibular second premolar.

esthetics. centric stability. chewing efficiency. soft tissue protection. none of the above.

In a lateral mandibular excursion, the teeth that ideally should provide the predominant guidance through the full range of the movement are the

1. 2. 3. 4. S.

1. 2. 3. 4. S. 6. 93.

a), (b) and (c) a) and (c) only b), (c) and (d) b) and (d) only cLr(d) and (e)

incisors. canines. premolars. incisors and canines. canines and premolars. incisors, canines and premolars.

98.

Which of the following ic;NOT characteristic of the permanent maxillary canine?

1. When viewed from the incisal, mesial


The combined mes;vdistal width of primary mandibular first and second molars, compared with the combined mesiodistal width 'of their permanent successors, is

2. Gingival pits and grooves, though not


3. 4. type traits, are common features.
.

and distal outl ines are asymmetrical.


When viewed from the facial, mesial and distal margins are parallel. The incisal margin occupies at least one-third of the crown height.

1. greater. 2. smaller. 3. equal.

99.
94.
If a maxillary first molar has a fourth canal, it is located in the

Which of the following is NOT a structural element of the temporomandibular joint?

1. 2. 3. 4.

fourth root. lingual root. distofacial root. mesiofacial root.

1. 2. 3. 4. S.

Sigmoid notch Condylar process Articular disk Capsular ligament Join t cavities

43

. .

NATIONAL BOARD DENTAL EXAMINATIONS PART I


ANSWER KEY

DENTAL ANATOMY -

DECEMBER 1979

No. -

Ans. -

Ans. No. 26. 27. 28. 29. 30. 31. 32. 33. 34. 35. 36. 37. 38. 39. 40. 41. 42. 43. 44. 45. 46. 47. 48. 49. 50. 4 2 1 4 2 5 3 1 5 3 2 2 4 1 7 3 2 4 2 3 5 2 2 2 4

No. Ans. 51. 52. 53. 54. 55. 56. 57. 58. 59. 60. 61. 62. 63. 64. 65. 66. 67. 68. 69. 70. 71. 72. 73. 74. 75. 1 2 3 4 3 1 1 2 3 4 3 2 4 5 3 2 1 2 1 3 3 1 1 2 4

No. 76. 77. 78. 79. 80. 81. 82. 83. 84. 85. 86. 87. 88. 89. 90. 91. 92. 93. 94. 95. 96. 97. 98. 99.

Ans. 2 4 2 4 4 4 5 2 3 2 1 1 1 3 1 4 2 1 4 1 4 2 3 1

1.
2. 3.

2
3 4 2 1 1 4 3 2 2 1 2 4 1 2 4 1 4 3 3 2 3 5 1 3

4.
5.

6. 7. 8.
9. 10. 11. 12. 13. 14. 15. 16. 17. 18. 19. 20. 21. 22. 23. 24. 25.

You might also like